btstudy.com 으로 오세요. 수능/내신 변형, 퀴즈를 무료로 공개합니다.

블루티쳐학원 | 등록번호: 762-94-00693 | 중고등 영어 | 수강료: 30(중등), 33(고등), 3+4(특강)

THE BLUET

728x90
반응형

20sw | Since 2005 임희재 | 블루티쳐학원 | 01033383436 | 200710 22:53:52

 

순서배열

 

1. 20수완 2-1

 

Adolescents' understanding of a task and the work necessary to complete it successfully influence their motivation.


(A) If a teacher assigns a chapter to read for homework without letting the students know that they are expected to discuss the major developments in the chapter the next day, then students do not understand the "real" assignment, nor do they know how to complete it successfully. 0

(B) For example, in assigning a textbook chapter for reading, the teacher should be clear about why the reading is assigned and what students are expected to do as a result of reading it. Provide guidance by giving examples of strategies that students can use in reading the chapter and relate that to successful participation in the discussion to enhance motivation for performing the reading activity. 2

(C) Goals and expectations for reading and writing assignments should be clear and specific. 1


① A-C-B ② B-A-C ③ B-C-A ④ C-A-B ⑤ C-B-A

 


2. 20수완 2-01

 

The understanding problem is simply that humans are not really set up to hear logic.


(A) We understand events in terms of other events we have already understood. When a decision-making heuristic, or rule of thumb, is presented to us without a context, we cannot decide the validity of the rule we have heard, nor do we know where to store this rule in our memories. Thus, the rule we are given is both difficult to evaluate and difficult to remember, making it virtually useless. 1

(B) People who fail to couch what they have to say in memorable stories will have their rules fall on deaf ears despite their best intentions, and despite the best intentions of their listeners. A good teacher is not one who explains things correctly, but one who couches his explanations in a memorable (i.e., an interesting) format. 2

(C) People, however, like to hear stories. The reason that people like to hear stories, however, is not transparent to them. People need a context to help them relate what they have heard to what they already know. 0


① A-C-B ② B-A-C ③ B-C-A ④ C-A-B ⑤ C-B-A

 


3. 20수완 2-02

 

Directness and honesty are qualities that our society values highly.


(A) Until we have good reason to know and trust the people behind the instant message pop-up or the chat room screen, we should remain anonymous and it is not dishonest to do so. 2

(B) We expect people to be who they say they are and tell us the truth about themselves. Before the advent of computers, anyone who used an assumed name was thought to be hiding something disreputable. 0

(C) Inexperienced computer users may continue to feel this way about online contacts. Because so many of the cues that we use to evaluate people are missing in cyberspace, computer users need to understand that virtual meeting places are different from face-to face contacts. 1


① A-C-B ② B-A-C ③ B-C-A ④ C-A-B ⑤ C-B-A

 


4. 20수완 2-03

 

To appreciate the long-term benefits of industrialization one does not have to accept its cruelties.


(A) Progress consists of unbundling the features of a social process as much as we can to maximize the human benefits while minimizing the harms. 2

(B) One can imagine an alternative history of the Industrial Revolution in which modern sensibilities applied earlier and the factories operated without children and with better working conditions for the adults. Today there are doubtless factories in the developing world that could offer as many jobs and still tum a profit while treating their workers more humanely. 0

(C) Pressure from trade negotiators and consumer protests has measurably improved working conditions in many places, and it is a natural progression as countries get richer and more integrated into the global community. Progress consists not in accepting every change as part of an indivisible package - as if we had to make a yes-or-no decision on whether the Industrial Revolution, or globalization, is a good thing or bad thing, exactly as each has unfolded in every detail. 1


① A-C-B ② B-A-C ③ B-C-A ④ C-A-B ⑤ C-B-A

 


5. 20수완 2-04

 

As for the advice to say what you said, the key is the expression "in other words."


(A) A summary should repeat enough of the key words to allow the reader to connect it back to the earlier passages that spelled out the points in detail. But those words should be fitted into new sentences that work together as a coherent passage of prose in its own right. 1

(B) There's no sense in copying a sentence from every paragraph and pasting them together at the end. That just forces the reader to figure out the point of those sentences all over again, and it is tantamount to a confession that the author isn't presenting ideas (which can always be clothed in different language) but just shuffling words around the page. 0

(C) The summary should be self-contained, almost as if the material being summarized had never existed. 2


① A-C-B ② B-A-C ③ B-C-A ④ C-A-B ⑤ C-B-A

 


6. 20수완 3-1

 

Because mobile phones are highly personal items (ie, they go with us everywhere and often are in constant contact with our bodies), many critics of wireless advertising (as well as advertisers themselves) are concerned that unwanted messages represent an invasion of privacy.


(A) In addition to privacy invasion, others are skeptical about wireless advertising's future on the grounds that advertising is antithetical to the reasons that people own mobile phones in the first place. 1

(B) The argument, in other words, is that people own mobile phones for reasons of enhancing time utilization and increasing work-related productivity while away from the workplace or home, and the last thing they want while using these devices is to receive unwanted, interrupting advertising messages. 2

(C) Feeling invaded, recipients of undesired advertisements may immediately delete the intruding item and hold negative feelings toward the offending advertiser. 0


① A-C-B ② B-A-C ③ B-C-A ④ C-A-B ⑤ C-B-A

 


7. 20수완 3-01

 

Marine debris affects animals through ingesting it or getting entangled in it; it is estimated that up to 100,000 marine mammals, including endangered species, are killed each year by marine debris.


(A) Large amounts of plastic debris have been found in the habitat of endangered Hawaiian monk seals, including in areas that serve as nurseries. Entanglement in plastic debris has led to injury and deaths in endangered Steller sea lions, with packing bands the most common entangling material. 1

(B) Very serious effects happen when marine animals become entangled in debris such as fishing line and six-pack rings. Birds get fishing line entangled around their legs, which get injured and may be lost. 0

(C) Hatchling sea turtles run down the beach to the ocean, a critical phase in their life cycle. Debris can be a major impediment if they get entangled in fishing nets or trapped in containers such as plastic cups and open canisters. Marine debris is an aspect of habitat quality for nesting sites and may help explain declines in turtle nest numbers on certain beaches. 2


① A-C-B ② B-A-C ③ B-C-A ④ C-A-B ⑤ C-B-A

 


8. 20수완 3-02

 

Regulations covering scientific experiments on human subjects are stringent.


(A) Human anatomy and physiology vary, in small but significant ways, according to gender, age, lifestyle, and other factors. Experimental results derived from a single subject are, therefore, of limited value. There is no way to know whether the subject's responses are typical or atypical of the response of humans as a group. 2

(B) Nonetheless, experimenting on oneself remains deeply problematic. One obvious drawback is the danger involved. Knowing that it exists does nothing to reduce it. A less obvious drawback is the limited range of data that the experiment can generate. 1

(C) Subjects must give their informed, written consent, and experimenters must submit their proposed experiments to rigorous scrutiny by overseeing bodies. Scientists who experiment on themselves can, functionally if not legally, avoid the restrictions associated with experimenting on other people. They can also sidestep most of the ethical issues involved. Nobody, presumably, is more aware of an experiment's potential hazards than the scientist who devised it. 0


① A-C-B ② B-A-C ③ B-C-A ④ C-A-B ⑤ C-B-A

 


9. 20수완 3-03

 

In many schools, the art program traditionally has been viewed as a particularly favorable setting for educating students with special needs.


(A) Children with hearing impairment can visually observe a demonstration of color mixing with paint and try the process with immediately verifiable results, and children with motor disabilities can work with finger paints or with large brushes for painting. 2

(B) In art classes, children are able to interact with such materials as paint or clay in direct response to their senses of sight, sound, smell, and touch. The materials of art are sensory, concrete, and manipulable in direct ways that are unique within the school curriculum. 0

(C) All the senses can be brought into interaction, providing opportunities to adapt art making activities for students who have some sensory or mobility impairment. For example, children with visual impairment can form expressive objects with clay. 1


① A-C-B ② B-A-C ③ B-C-A ④ C-A-B ⑤ C-B-A

 


10. 20수완 3-04

 

Medical imaging techniques are constantly being refined, and this effort has been aided by various space technologies over the years.


(A) Digital image processing techniques developed at NASA's Jet Propulsion Laboratory to allow for computer enhancement of lunar pictures from the Apollo missions have since led to improved MRI and CT imaging. Techniques in astronomy have also refined imaging. 0

(B) Moreover, the procedure is ten times cheaper than a surgical biopsy, and greatly reduces the pain, scarring, radiation exposure and time associated with surgical biopsies. 2

(C) The very same infrared sensors used to remotely observe the temperature of stars and planets are now being used to help surgeons map brain tumors. Charge-Coupled Device chip technologies stemming from the Hubble Telescope have greatly furthered breast cancer detection techniques, allowing breast tissue to be imaged more clearly and efficiently, thus increasing resolution so as to be able to distinguish between malignant and benign tumors without resorting to surgical biopsy. 1


① A-C-B ② B-A-C ③ B-C-A ④ C-A-B ⑤ C-B-A

 


11. 20수완 4-1

 

As the health of the social body began to change, hospitals became the primary institutions to care for the sick and monitor the passage from life to death.


(A) The dramatic rise in the number of hospitals across the country and their increasing control over the health of the nation contributed to the separation of death from everyday life. As doctors achieved professional dominance in the practice of medicine in the early decades of the twentieth century, hospitals emerged as the principal site for the diagnosis and treatment of patients. 0

(B) An 1873 survey counted 178 hospitals, about 50 of which were institutions for the mentally ill. A 1923 tabulation listed 6,830, or an increase of about 3,800 percent. While this change was most dramatic in the urban landscape, it also occurred gradually in rural areas. It should be no surprise that one consequence was an increase in the number of deaths away from home, the traditional place for end of life scenarios. 2

(C) In the words of one social historian, Whereas doctors came to patients in the 1870s, by the 1920s, patients increasingly came to doctors. Over this time span, the American hospital changed in size and clientele. 1


① A-C-B ② B-A-C ③ B-C-A ④ C-A-B ⑤ C-B-A

 


12. 20수완 4-01

 

The way in which people shape landscapes often reflects the dominant culture.


(A) In a highly commercialized setting such as the United States, it is not surprising that many landscapes are seen as commodities. In other words, they are valued because of their market potential. 0

(B) The landscape itself, including the people and their sense of self, takes on the form of a commodity. Over time, the landscape identity can evolve into a sort of "logo" that can be used to sell the stories of the landscape. Thus, California's "Wine Country," Florida's "Sun Coast," or South Dakota's "Badlands" shape how both outsiders and residents perceive a place, and these labels build a set of expectations associated with the culture of those who live there. 2

(C) Residents develop an identity in part based on how the landscape can generate income for the community. This process involves more than the conversion of the natural elements into commodities. 1


① A-C-B ② B-A-C ③ B-C-A ④ C-A-B ⑤ C-B-A

 


13. 20수완 4-02

 

There are multiple views of the learner in theories of cognition, just as there are multiple theories of the nature of art.


(A) Some portray the learner as a lone individual trying to make sense of a work of art or, for that matter, the world. Others picture learners as living within a social or cultural context from which knowledge derives its meaning. 0

(B) But if works of art are thought of as autonomous structures whose meaning is set by the artist, where there is one "objective" or "right" interpretation, independent of its social context, then it might make more sense to pair this conception of art with the view of the learner as a lone individual. 2

(C) Such different views of the learner have consequences for teaching the arts. At the same time, if works of art are to be understood in terms of their social and cultural origins and purposes, then it would make sense to integrate the knowledge of the artwork into those subjects, such as the social studies or history, where knowledge of the culture and society is collaterally provided. 1


① A-C-B ② B-A-C ③ B-C-A ④ C-A-B ⑤ C-B-A

 


14. 20수완 4-03

 

You may resist the notion that music, which you think should be free to express emotion, is best thought of as rule-governed behavior.


(A) But rules govern all meaningful human cultural behavior in just this way. The point is not that musical performance is predetermined by rules, but that it proceeds according to them. 0

(B) Nonetheless, just as meaningful conversations can express emotion, so meaningful music can express it as well, though not, of course, in exactly the same way. Further, if a listener does not understand the rules, he or she can understand neither the intention of the composer or musician nor the music's structure. 2

(C) In this view, music is like a game or a conversation. Without rules we could not have a game, and without agreement about what words are, what they mean, and how they are used, we could not hold a meaningful conversation. 1


① A-C-B ② B-A-C ③ B-C-A ④ C-A-B ⑤ C-B-A

 


15. 20수완 4-04

 

Since the 19th century, economics has borne the dishonorable name of "the dismal science."


(A) But Carlyle's negative characterization of economics in fact appears in an article that he wrote about slavery in the West Indies, not in any of his writings about Malthus. And, as it turns out, Carlyle delivered his insult to economics simply because the free market economists of his time did not support his proslavery views. So, in the end, economics earned its less than auspicious nickname for being on what most would agree was the right side of history. 2

(B) The cheerless nickname has been attributed by some to the Scottish historian Thomas Carlyle, who reportedly coined the term when discussing economist Thomas Malthus's prediction that one day population growth would outstrip food production and cause widespread famine. It's true that Carlyle wrote about Malthus on occasion. 1

(C) Some people think economics is called "the dismal science" because it's a dry and difficult subject. Others think it's because economics tackles depressing topics such as poverty, crime, war, taxes, inflation, and economic collapse. 0


① A-C-B ② B-A-C ③ B-C-A ④ C-A-B ⑤ C-B-A

 


16. 20수완 8-1

 

The abundant supply of commercial fertilizers has made possible the production of large crops on land that was once considered "worn out".


(A) This applies particularly to phosphorus, much of which is chemically fixed in the soil; the same is true to a lesser extent of nitrogen and potassium. Heavy fertilization that results in large yields also commonly slowly increases the soil organic matter content if the soil was very low in it initially, and necessarily the nitrogen content of the soil. 1

(B) It has also led to improved practices with regard to drainage, erosion control, and many other practices because fertilizers make it profitable to spend money on soils to put them in the best possible condition for high yields. Not all of the added fertilizer is removed by the first one or two crops but some of it remains fixed in the soil in a slowly available form. 0

(C) The result then is that fertilizers tend to increase soil fertility, or at least soil productivity. 2


① A-C-B ② B-A-C ③ B-C-A ④ C-A-B ⑤ C-B-A

 


17. 20수완 8-01

 

Communal living became a necessity because the community and the division of labour through which all individuals subordinate themselves to the group, ensured the continued existence of the species.


(A) Consider the difficulties of childbirth and the extraordinary care necessary for keeping a child alive during its infancy! This prolonged care and attention could only be exercised where division of labour existed. 1

(B) Think of the number of illnesses and disorders that human flesh is heir to, particularly in infancy, and you have some conception of the inordinate amount of care each human life demands, and some understanding of the need for communal living. The community is the best guarantor of the continued existence of human beings! 2

(C) Only division of labour (which is another way of saying civilization) is capable of ensuring that the tools of survival are available to humankind. Only after they had learned about the division of labour did humans learn how to assert themselves. 0


① A-C-B ② B-A-C ③ B-C-A ④ C-A-B ⑤ C-B-A

 


18. 20수완 8-02

 

Remember the Great Blackout of 2003?


(A) But some do. For example, back in February 2000, cyberpranksters launched the first major attack of the Internet Age. Their targets included the biggest of the big online operators. The weapon in this case was a "denial of service" attack where attackers bombard a target's servers with thousands of hits. 2

(B) A glitch in the US electrical grid knocked out power in a huge swath stretching from New York City to the Midwest. Similar incidents struck Italy and Norway in 2003. Technical breakdowns that happen on a grand scale also happen on a small scale to individual businesses. Everyone knows what it's like when the company server goes down. 0

(C) You cannot send or receive e-mail. Access to your databases evaporates. Customers cannot place orders on your Web site. Short interruptions, of course, rarely constitute a crisis. 1


① A-C-B ② B-A-C ③ B-C-A ④ C-A-B ⑤ C-B-A

 


19. 20수완 8-03

 

In 1843, German economist Wilhelm Roscher explicitly raised the issue of the need to use the historical method in economics, the essential element of which should be a comparative approach.


(A) Roscher argued that economic behaviours are dependent on their historical and social context, causes that in their studies should be used not only in an economic but also a historical and sociological approach. 0

(B) This explains why so much effort of the representatives of the so-called older historical school, notably Bruno Hildebrand and Karl Knies, focused on describing the stages of the historical development of society. There is often talk of the methodological assumption common to most representatives of this school of thought, shared by their English counterparts, which was a denial of the existence of universally valid economic laws in conjunction with the emphasis on the importance of individual facts. 2

(C) The first task of a researcher is therefore to immerse oneself in history to get as wide a knowledge of economic facts as possible, and on that basis determine the relationship between the economy and society. 1


① A-C-B ② B-A-C ③ B-C-A ④ C-A-B ⑤ C-B-A

 


20. 20수완 8-04

 

A guarantee of authenticity through a form of certification is important to many tourist consumers.


(A) The same study also noted that domestic tourists from New Zealand, as well as visitors from Australia and Asia, considered verification of genuineness on labels very important in their purchasing decisions, while visitors from Europe and North America were less concerned with the issue. 2

(B) Documentation of legitimacy may be provided in the form of official paperwork and certificates, photographs, artisans' signatures, and dates. In their 1993 paper, Littrell and her colleagues suggest that it is typically tourists with a need for status who often judge authenticity by external markers such as these. 0

(C) According to nearly half of the visitors in one New Zealand study, authenticity was important in their decision to buy an item of clothing. When asked if they would be more likely to buy clothing from New Zealand if design authenticity were included on the label, 46% said that they would. 1


① A-C-B ② B-A-C ③ B-C-A ④ C-A-B ⑤ C-B-A

 


21. 20수완 9-1

 

A paradoxical effect of better communications has been social fragmentation and isolation.


(A) Today there are many new ways in which people are able to lead rich and fulfilling social lives. And yet the social fabric that connected people at the local, geographical level has largely disappeared. And the trend towards social fragmentation just keeps on increasing. Many technological changes were introduced in the name of efficiency and time-saving for the consumer.1

(B) It is more convenient if you can do your banking online from home. But the unforeseen side effect is to chip away at our exposure to social contact. Perhaps the most serious consequence is that convenient technologies are fragmenting the social networks that reinforce important moral, ethical and social values. 2

(C) Social contact is a fundamental human need. We are social animals. Technological innovation has contributed relentlessly to the isolation of people from one another. As the increasing frequency of people dying alone reveals, all is not well in modern society. 0


① A-C-B ② B-A-C ③ B-C-A ④ C-A-B ⑤ C-B-A

 


22. 20수완 9-01

 

While humor is one of the important coping mechanisms of Native Americans, it should be used only if the client invites it, meaning that the client trusts the counselor enough to connect on that level.


(A) Indian humor serves the purpose of reaffirming and enhancing the sense of connectedness as part of family, clan, and tribe. To the extent that it can serve that purpose in the counseling relationship, it is all the better. 2

(B) What, in one situation, can be humor between two people, in another can be interpreted as ridicule or wearing a mask. Counselors, therefore, have to be sensitive to using humor in a way that doesn't reinforce various means of oppression that the client has endured probably for all of her or his life. 0

(C) However, in the positive direction, humor provides the opportunity to connect with the client on her or his ground and share a powerful trust. In sum, although counselors working with Native American clients should exercise caution when using humor, they definitely should not overlook it as a powerful therapeutic technique. 1


① A-C-B ② B-A-C ③ B-C-A ④ C-A-B ⑤ C-B-A

 


23. 20수완 9-02

 

For millions of years man and his closest ancestors obtained food (and medicinal substances) from nature in a ready form.


(A) The sharp fall in rural populations that accompanied the growth of cities and the development of so-called public catering necessitated the production and storage of enormous quantities of food products. This coincided with the emergence of a second way of deriving material benefits - 'production'. But side by side with production, in the sense of the creation of completely new objects for consumption, goes the process of purification (distillation, refinement, etc.). 1

(B) We have called this process 'anti-gathering'. It has led to finely ground flour products, polished rice and other grain removed from the husk, refined oil, and refined sugar. The choice of vegetables and wild-growing plants has shrunk; pure spirits such as vodka have replaced natural wines. 2

(C) There was nothing to prevent the whole diversity of biologically active substances from entering his body. Cooking food was an individual domestic affair. 'Gathering' was the first way man received material benefits. 0


① A-C-B ② B-A-C ③ B-C-A ④ C-A-B ⑤ C-B-A

 


24. 20수완 9-03

 

In the offline world, bystander helping is influenced by the number of other people available to provide help.


(A) The combination of visible needs for help and unknown numbers of potential helpers may make the felt need to offer help more salient. Until one person actually offers help, every potential helper may assume that he or she is the only one who could help. 1

(B) Physical invisibility also reduces the barriers to offering help for people whose age, gender, race, or other visible attributes lead people to discount their contributions in the offline world, regardless of their actual usefulness. Help provided is not spumed on the basis of irrelevant physical or social attributes; it can be judged based solely on its quality. 2

(C) If people see that others are available to help, their own motivation to help is diminished. In the online world, it is hard to know how many potential helpers are available. 0


① A-C-B ② B-A-C ③ B-C-A ④ C-A-B ⑤ C-B-A

 


25. 20수완 9-04

 

Fear of strangers (xenophobia) has had a long history.


(A) They do not share our history, our beliefs or ideology, our religion or our work ethic so they are unwelcome. They do not have to be afforded equal consideration in the application of our values or laws. Racism and prejudice are passed along from generation to generation in a process known as cultural transmission. 1

(B) Foreigners, an out-group, were seen as threatening simply by being different. Immigration policies have continued to reflect this concern that foreigners use up our resources, take our jobs, and become a burden or threat to society. People from other lands and other groups gain a negative stereotype. 0

(C) Primary groups reinforce prejudices and stereotypes during socialization. The media disproportionately report negatives about minorities (out-groups), institutions advance discriminatory practices, and governments institute programs and policies that deny equal opportunity. As these become entrenched, they are difficult to challenge. 2


① A-C-B ② B-A-C ③ B-C-A ④ C-A-B ⑤ C-B-A

 


26. 20수완 10-1

 

The history of virtue ethics goes back to Aristotle in The Nicomachean Ethics.


(A) This leads to circularity, to a life of purposelessness. Take the following example. I go to work to earn money. I earn money to enable me to purchase food. I purchase food so that I can eat. 1

(B) I eat in order to go to work. I work to earn money. According to Aristotle, the purpose of all our acts is that they are directed to some ultimate good. We should seek to know the good so that we are able to direct our actions to that end. Without trying to know the good we avoid the fundamental purpose for human life. 2

(C) As in the rest of his philosophy, the notion of purpose (goal, ends) plays an integral role in Aristotle's ethical theory. Every act, he says, is performed for some purpose. He defined this purpose as 'the good'. In other words, we do the things we do because they have a worthwhile purpose. Otherwise, the acts we perform would depend on some other act to give them meaning and so forth. 0


① A-C-B ② B-A-C ③ B-C-A ④ C-A-B ⑤ C-B-A

 


27. 20수완 10-01

 

If the hypothesis under consideration is a simple generalization, it may be sufficient to test it by looking for more examples, seeing whether or not the generalization holds for them.


(A) If the generalization has any reasonable body of supporting data, the finding of new facts which do not fit usually leads to the refinement or elaboration of the original hypothesis rather than its complete rejection. 1

(B) On the other hand, if the original basis for the hypothesis was slender, the unfavorable instances may so outweigh the favorable ones as to make it reasonable to believe that the earlier agreement was a matter of pure chance. Also, a new hypothesis may be developed which fits the original data and the new data as well. 2

(C) Under these circumstances unfavorable examples which violate the generalization may or may not lead to its rejection. 0


① A-C-B ② B-A-C ③ B-C-A ④ C-A-B ⑤ C-B-A

 


28. 20수완 10-02

 

Knowledge — the output of human innovation — is unique among all resources.


(A) It's not a physical resource. It's an information resource. Where all physical resources are depleted by use, and are divided by sharing, knowledge is different. 0

(B) That means the world isn't zero-sum. One person or nation's gain doesn't have to be another's loss. By creating new ideas, we can enrich all of us on the planet, while impoverishing none. Knowledge plays by different rules than physical resources, rules that make it inherently abundant. 2

(C) A wheel may break or wear out, but the idea of the wheel will keep on working. A wheel can only be used in one place and one time, but the design for a wheel can be shared with an infinite number of people, all of whom can benefit from it. Ideas aren't zero-sum. 1


① A-C-B ② B-A-C ③ B-C-A ④ C-A-B ⑤ C-B-A

 


29. 20수완 10-03

 

To at least "compare notes" with other philosophically interested people adds something crucial to the practice of philosophy itself and not just because it's important to make your ideas more widely known.


(A) Rather, it helps you hone your ideas to greater quality. Part of this is making sure that your ideas and reasoning process are as clearly spelled out as possible; ensuring that they're clear to someone else helps make them clearer to yourself. 0

(B) This gives both participants in a philosophical exchange an indispensable opportunity to hone their ideas and reasoning skills. Thus, the social component of philosophy is an essential part of the doing of philosophy itself. 2

(C) This is how a philosopher ensures that he or she is really making sense and using valid reasoning processes. If you are challenged by someone who seems to disagree, this forces you to be as clear as possible about what principles you are starting with and exactly how you are reasoning to a given conclusion. 1


① A-C-B ② B-A-C ③ B-C-A ④ C-A-B ⑤ C-B-A

 


30. 20수완 10-04

 

In a formalized manner the social structure of the indigenous peoples of the Pacific Northwest Coast is based on intricate forms of clan and family lineage regulated by both birth and wealth.


(A) In this way the ownership and display of ritual clothing was literally the visual fabric of the social order. 2

(B) Rank and privilege were proclaimed by song, speech, and costume in tribal cultures where relative status in the community was of prime importance. Whole costumes from headgear to leggings were decorated with a readable heraldic system based on familiar animals associated with clan symbolism, and personal, often inherited, spiritual relationships. 0

(C) Costumes were worn at the social and religious events that expressed their highly differentiated and formally structured society. As a new rank was achieved or a fresh honor bestowed upon an individual, the concurrent rise in prestige and status was often marked by that person's right to wear another highly specialized symbolic garment. 1


① A-C-B ② B-A-C ③ B-C-A ④ C-A-B ⑤ C-B-A

 


31. 20수완 11-1

 

Unexpected events typically lead us to think in more complex ways.


(A) Subjects in one study read about a student who had done either well or poorly in high school and then learned about the student's college grades. For some of the subjects, their expectations were confirmed. For example, the good student in high school received good grades in college. 0

(B) Subjects who learned of the unexpected outcome considered many more causal attributions (eg, "perhaps he did much better than expected because he finally learned how to study") than subjects who simply had their expectation confirmed. This study demonstrates that unexpected events increase our search for explanations. 2

(C) For others, their expectations were violated. For example, the poor student in high school did unexpectedly well in college. Subjects then retold the story into a tape recorder as if they were relaying it to a friend. 1


① A-C-B ② B-A-C ③ B-C-A ④ C-A-B ⑤ C-B-A

 


32. 20수완 11-01

 

There is an old television commercial from the seventies that shows a Native American walking along a polluted river.


(A) Every action has a reaction. You may not realize it today, but someone will in the future. 2

(B) If we pull up our stakes and move the family to Florida, they will grow up as Southerners and not as New Englanders. They will live in a world devoid of snow and cold and be reliant on fans and air conditioners for their comfort. If you work eighteen hours a day and your children are raised by babysitters and other child care providers, do not be surprised at the people they grow up to be. 1

(C) The garbage floats to the top and onto the river banks as a tear flows down his cheek. There are no words spoken by the man but it is clear that he is ashamed and appalled at what we have done to the beautiful land that was once his ancestors'. Every day, we fail to realize that our actions have an impact on the future. 0


① A-C-B ② B-A-C ③ B-C-A ④ C-A-B ⑤ C-B-A

 


33. 20수완 11-02

 

In a series of experiments by Jacques Mehler and his colleagues, infants as young as four days old were able to distinguish their native language from a different language, while they were unable to distinguish utterances in two foreign languages.


(A) The babies were more aroused by utterances in the native language, as indicated by the faster rate at which they sucked on their pacifiers. On the basis of several studies indicating that some sound from speech reaches infants in utero, although reduced in frequency range and intensity, Mehler and his colleagues tested very young infants with highly filtered versions of recordings in the native language and one that was nonnative. 0

(B) The experimenters conclude that prosody is sufficient for infants to discriminate the two languages. 2

(C) The infants were able to discriminate preferentially in favor of their native language. This suggests that prosodic cues play an important role in the infants' responses, since those were the only cues available on the filtered tapes. 1


① A-C-B ② B-A-C ③ B-C-A ④ C-A-B ⑤ C-B-A

 


34. 20수완 11-03

 

We sometimes solve number problems almost without realizing it.


(A) As long as you do not run out of copies before completing this process, you will know that you have a sufficient number to go around. You have then solved this problem without resorting to arithmetic and without explicit counting. 1

(B) There are numbers at work for us here all the same and they allow precise comparison of one collection with another, even though the members that make up the collections could have entirely different characters, as is the case here, where one set is a collection of people, while the other consists of pieces of paper. What numbers allow us to do is to compare the relative size of one set with another. 2

(C) For example, suppose you are conducting a meeting and you want to ensure that everyone there has a copy of the agenda. You can deal with this by labelling each copy of the handout in turn with the initials of each of those present. 0


① A-C-B ② B-A-C ③ B-C-A ④ C-A-B ⑤ C-B-A

 


35. 20수완 11-04

 

For more than two decades, school reform has been driven by an agenda that appears to be uninformed by even the most basic research into what we now know about the functioning of the brain or the healthy development of the child.


(A) Educational leaders and policy makers aren't asking, "What do children need for healthy brain development?" "How do they learn best?" 0

(B) Most reform has been focused on what to cram into children's heads — and testing them ad nauseam to see what sticks — rather than on developing their brains. What this means is that we do more but accomplish less. Educational reforms fail because they hinge on policies that lower the sense of control of students, teachers, and administrators alike, predictably leading to greater stress, lower student engagement, and ever more teacher dissatisfaction and burnout. 2

(C) or "When's the optimal time to teach him or her to read or do algebra?" Rather, they seem to be asking, "What do we need this child to be able to do in order to meet our school, local, or national standards?" 1


① A-C-B ② B-A-C ③ B-C-A ④ C-A-B ⑤ C-B-A

 


36. 20수완 12-1

 

What Mark Twain achieves, with great skill and at tremendous risk, all the way through his book Huckleberry Finn, is an invisible but immense ironic distance between his point of view and Huck's.


(A) Huck might grow up into that kind of man, given a chance. But Huck at this point is an ignorant, prejudiced kid who doesn't know right from wrong (though once, when it really matters, he guesses right). In the tension between that kid's voice and Mark Twain's silence lies much of the power of the book. We have to understand — as soon as we're old enough to read this way — that what the book really says lies in that silence. 2

(B) Huck tells the story. Every word of it is in his voice, from his point of view. Mark is silent. 0

(C) Mark's point of view, particularly as regards slavery and the character Jim, is never stated. It is discernible only in the story itself and the characters — Jim's character, above all. Jim is the only real adult in the book, a kind, warm, strong, patient man, with a delicate and powerful sense of morality. 1


① A-C-B ② B-A-C ③ B-C-A ④ C-A-B ⑤ C-B-A

 


37. 20수완 12-01

 

Think of life as a charity foot race.


(A) Winning can be enjoying the scenery, the nice people along the way, or even the time for reflection during the relaxed race. Remember the old rat race analogy — it might be more important not to be a rat than to win the rat race. After all, if you win the rat race, you will still be a rat. 2

(B) The race, like many things in life, will have a similar outcome for most of the participants —they will all reach the finish line. The only difference then, is the process of running the race. What then is the difference between the person, eyes focused straight ahead, who rushes from the starting line to the finish line in a wild dash hope of winning and the person who looks to the right and left and still arrives at the finish line? 0

(C) Not that there's anything wrong with winning races. However, only one person can win by reaching the finish line first, while everyone can win from the standpoint of the process. The key is to develop a personal definition of winning. 1


① A-C-B ② B-A-C ③ B-C-A ④ C-A-B ⑤ C-B-A

 


38. 20수완 12-02

 

For many people, the words "Cajun" and "Creole" lead to visions of gumbo, red beans and rice, crawfish, and just about anything that's been "blackened, Cajun-style."


(A) While these culinary traditions are distinctive and delicious, they have overshadowed the many other unique cultural contributions made by Louisiana's Cajun and Creole communities, and are often considered without reference to the social and historical contexts that produced them. 0

(B) As poet Sheryl St. Germain writes in "Cajun," she fears "the word's been stolen" by retail commodifiers of a culture emptied of its content and its history, reduced — quite literally - to an object of consumption. 2

(C) When "Cajun" and "Creole" are reduced to adjectives on menus and food labels, it becomes easy to lose sight of the people those adjectives first described. 1


① A-C-B ② B-A-C ③ B-C-A ④ C-A-B ⑤ C-B-A

 


39. 20수완 12-03

 

As Larry Cuban, former president of the American Educational Research Association, points out, schools have spent billions of dollars over the years on technologies that have, in fact, changed very little of how we think about an "education" in the developed world.


(A) In fact, the system almost unwittingly marginalizes digital technologies in schools. We relegate them to labs or libraries, or if we place them in students' hands, they're used only for discrete, narrow purposes like reading textbooks, creating documents, or taking assessments. 1

(B) More often than not, we strip the agency and freedoms that digital tools give to learners and creators outside of school when they bring those same tools into the building. The system of schooling that most of us are products of is based on a series of structures and efficiencies that do not work well with the messier, less linear, more self-organized ways we can learn, create, and connect on the Internet. 0

(C) Few would argue that in schools today, we see technology primarily as an institutional teaching tool, not a personal learning tool. 2


① A-C-B ② B-A-C ③ B-C-A ④ C-A-B ⑤ C-B-A

 


40. 20수완 12-04

 

It is pretty well known that creativity functions best with periods of incubation, but it can be really hard to integrate this nothing time into your work practice.


(A) But sleep has been shown to be a very significant part of the creative process which utilizes intuition and insight and which leads you down a much more successful path to problem solving. Therefore, this proves that not only is nothing actually something, but it is in fact crucial. 2

(B) I've always felt a bit guilty about my regular half-hour afternoon naps, partly because I'm so conscious that I'm not doing anything and it often feels more like I'm escaping. 1

(C) However, it is vital unconscious time when our minds are able to rework elements or problems that have arisen through conscious activity and which, more often than not, result in new and fresh ideas. 0


① A-C-B ② B-A-C ③ B-C-A ④ C-A-B ⑤ C-B-A

 


41. 20수완 13-1

 

Licenses in copyright work similarly to other licenses that we deal with in everyday life; they give a person or entity permission to do certain acts within certain limits.


(A) When granting a copyright license, the scope of the license may also be bounded by time or by the type of licensee to whom it is granted. It may be restricted by the types of use the licensee may make of the work or by the parts of the bundle of rights granted. 2

(B) These limits are the scope of the license and define which acts are permissible. 0

(C) For example, a person may have a license to practice law in one state but not in another state, and that license to practice law does not also give the licensee a driver's license. 1


① A-C-B ② B-A-C ③ B-C-A ④ C-A-B ⑤ C-B-A

 


42. 20수완 13-01

 

The rising challenge today for library workers is to help users not only find information on the Internet, but also become skillful evaluators of its usefulness and reliability from the ocean of resources available.


(A) Staff members in special libraries have a responsibility to sift through and evaluate data for their colleagues. For public librarians and staff, helping users find and evaluate information is central to their mission. 1

(B) In all cases, the library worker's role as an evaluator is just as important as that of a facilitator. For these reasons, libraries remain alive and well, because the Internet complements libraries, but does no replace them. 2

(C) Critical thinking skills are also essential. In school and academic libraries, this responsibility is shared with classroom teachers. 0


① A-C-B ② B-A-C ③ B-C-A ④ C-A-B ⑤ C-B-A

 


43. 20수완 13-02

 

Mathematics is one of the most profound creations of the human mind.


(A) How many of the greatest minds in history, from Pythagoras to Galileo to Gauss to Einstein, have held that "God is a mathematician"? 1

(B) For thousands of years, the content of mathematical theories seemed to tell us something profound about the nature of the natural world — something that could not be expressed in any way other than the mathematical. 0

(C) This attitude reveals a reverence for mathematics that is occasioned by the sense that nature has a secret code that reveals her hidden order. The immediate evidence from the natural world may seem to be chaotic and without any inner regularity, but mathematics reveals that under the surface the world of nature has an unexpected simplicity — an extraordinary beauty and order. 2


① A-C-B ② B-A-C ③ B-C-A ④ C-A-B ⑤ C-B-A

 


44. 20수완 13-03

 

Sociologist Harry Edwards argues that both sport and religion have a body of formally stated beliefs that are accepted on faith by a great number of adherents.


(A) Belief systems are centered on a creed that followers abide by in varying degrees. 0

(B) The philosopher William James coined the phrase "the will to believe" to express the view that sometimes believing that an event will occur can actually help make it happen. This can be applied to both athletes and fans, who feel that their strong dedication to winning might actually tip the scale and make victory occur. 2

(C) Fans are told to have "faith" in their team, especially in times of turmoil, much in the same manner that religious people are told to have faith in times where they doubt their religion. 1


① A-C-B ② B-A-C ③ B-C-A ④ C-A-B ⑤ C-B-A

 


45. 20수완 13-04

 

The advent of technological innovations in sound production has had radical (some might argue liberating) consequences for film scoring and recording.


(A) It is theoretically possible now, in many parts of the world, for a composer to virtually create and produce an entire score, thus eliminating the need for teams of assistants, arrangers, and copyists to realize the score, and live musicians on acoustic instruments to perform it. 0

(B) In Bollywood, for instance, it has become so commonplace for scores to be digitally produced on a synthesizer that the very nomenclature has begun to change, with the term "programmer" replacing "music director." Such changes to film scoring have been so dramatic and the economic consequences so profound that Gregory Booth, in a recent book on the Mumbai film industry, labels the preprogramming years "Old Bollywood" and postprogramming "New Bollywood." 2

(C) In many film industries it is becoming increasingly necessary for composers to have computer expertise. 1


① A-C-B ② B-A-C ③ B-C-A ④ C-A-B ⑤ C-B-A

 


46. 20수완 14-1

 

One point of difference between the consumption of water and electricity is that water can be reused multiple times while electricity cannot.


(A) Withdrawn water, on the other hand, can be returned to its original water source. The argument can be made that all water demand eventually returns as precipitation via the hydrologic cycle and therefore is not "consumed". 1

(B) As a result, water can be classified as "consumed" or simply "withdrawn". In the former, water is removed from its source and lost through either evaporation (in the case of power plant cooling or flood irrigation), or transpiration (in the growing of biocrops). 0

(C) However, evaporation and precipitation are both spatially and temporally uneven. Water that is accessible, especially in arid and semi-arid regions, satisfies the immediate needs of water users, whereas future precipitation may not occur in the same location or at the desired timing. 2


① A-C-B ② B-A-C ③ B-C-A ④ C-A-B ⑤ C-B-A

 


47. 20수완 14-01

 

Today, Luddite is a disparaging term used to refer to a person who is opposed to or cautiously critical of technology.


(A) But it's important to remember that the original Luddites were not, in fact, opposed to technology per se. It was not the machines themselves that the Luddites feared and reacted against. Rather, they understood that technology is meant to serve humans, not the other way around. 0

(B) Suddenly they were answerable not to themselves but to a factory owner; they had to give up autonomy, or starve. They saw what the machines meant to their livelihood, to their lives, to their families, and to their communities. And they didn't like what they saw. 2

(C) Luddites were not protesting the technology itself; they were objecting to the new economic realities brought about by the machines. In former times, craftsmen had been able to work at their own pace and set their own prices for their goods. But with the dawn of industrialization and mass production, craftsmen fell on hard times and were increasingly forced to work for the hated factories. 1


① A-C-B ② B-A-C ③ B-C-A ④ C-A-B ⑤ C-B-A

 


48. 20수완 14-02

 

The free market will tend to oversupply goods with negative externalities, and undersupply goods with positive externalities.


(A) Public goods, on the other hand, are subject to the free-rider problem. Why should I contribute to supply street lights if I will get the benefit whether or not I contribute? But if everyone thinks like this — and the market encourages this type of reasoning — no lighting will be provided. 1

(B) It is easy to see why. Creating a negative externality is often a way of dumping your costs on another: literally. If it is cheaper to use a noisy production process than a quiet one, other people are inadvertently 'subsidizing' my use of the noisy process by bearing the cost of being disturbed by the noise. 0

(C) It is normally assumed that the solution to these problems is to make the state the supplier of public goods, taxing citizens to pay for them. Similarly the state can make pollution illegal, returning the costs to the polluter. 2


① A-C-B ② B-A-C ③ B-C-A ④ C-A-B ⑤ C-B-A

 


49. 20수완 14-03

 

The September 11, 2001, terrorist attacks left the economy reeling from a lack of spending and confidence.


(A) One industry that was affected the most was the airline industry. This industry suffered huge and insurmountable losses that threatened the survival of the largest airline companies. Without air travel, businesses became slower, less efficient, and less reliable. In this instance, a lack of demand wasn't necessarily indicative of consumers' true feelings toward air travel. 0

(B) The answer depended on time. If the government had stood by and let the economy regulate itself in this situation, the airline industry may have recovered, but in the process, countless more jobs would have been lost. So the government intervened and put together a monetary aid package to keep the airline industry alive. 2

(C) Rather, this lack of demand was created by an extreme disaster. So the question became, what should be done? Should the government have let the forces of supply and demand take over and eventually revamp the airline industry? Or should the government have taken a more proactive approach? 1


① A-C-B ② B-A-C ③ B-C-A ④ C-A-B ⑤ C-B-A

 


50. 20수완 14-04

 

While time is being compressed by the frantic pace of modern life, our sense of space has expanded to the point where the concept of place is no longer salient.


(A) While democratic in appearance, in reality these spaces are designed to coordinate economic activity across nodes in the global commodity chain, obedient to the protocols of external control, with every public space given over to marketing the same products to consumers seduced by ahistorical, transregional brands created by the advertising industry. 1

(B) Most public spaces are now commercial spaces standardized to provide comfort zones and facilitate easy access and mobility by anyone who might enter - a famous coffee shop on every corner, with vast swathes of every city resembling an airport terminal. 0

(C) In the process, genuinely local places are stripped of uniqueness and particularity. Each location resembles every other location, and only the occasional extraordinary effort on the part of the locals preserves a genuine sense of place. 2


① A-C-B ② B-A-C ③ B-C-A ④ C-A-B ⑤ C-B-A

 


51. 20수완 15-1

 

Businesses that own enterprises and seek to market their goods and services in more than one nation are known as multinational corporations (MNCs).


(A) Poor nations are particularly vulnerable to MNCs, many of which have yearly budgets greater than those of poor-nation governments. For example, in 2014, each of the world's 20 largest MNCs had gross revenues of more than $161 billion, larger than all but about 50 of the 208 countries tracked by the World Bank. 1

(B) MNCs bring employment opportunities as well as goods and services to people who otherwise would not have them. At the same time, they create major and controversial changes in the natural, economic, social, and political environments. 0

(C) The financial power of these corporations enables them to exert enormous influence on poor nations and makes it extremely difficult for these nations to regulate them. In addition, like all capitalist corporations, the fundamental goal of MNCs is to return wealth to their shareholders, the vast majority of whom live in wealthy nations. Thus, most MNC profits in poor nations contribute to the economy of wealthy nations. 2


① A-C-B ② B-A-C ③ B-C-A ④ C-A-B ⑤ C-B-A

 


52. 20수완 15-01

 

In the popular media, archaeology is mainly identified with spectacular discoveries of artifacts from prehistoric and ancient cultures, such as the tomb of the Egyptian king Tutankhamun.


(A) Their principal task is to infer the nature of past cultures based on the patterns of the artifacts left behind. Archaeologists work like detectives, slowly sifting and interpreting evidence. 1

(B) The context in which things are found, the location of an archaeological site, and the precise position of an artifact within that site are critical to interpretation. In fact, these considerations may be more important than the artifact itself. 2

(C) As a result, people often think of archaeologists primarily as collectors. But contemporary archaeologists are much more interested in understanding and explaining their finds in terms of what those objects say about the behavior that produced them than in creating collections. 0


① A-C-B ② B-A-C ③ B-C-A ④ C-A-B ⑤ C-B-A

 


53. 20수완 15-02

 

Violence is common among many living organisms and probably has roots that stretch back to the time of the first noncloned life forms that were capable of physical interaction.


(A) "Violence" is defined as one or more individuals physically attacking one or more other individuals. The term "violence" is typically used in a negative, antisocial context, especially regarding humans. 1

(B) The term "violence" is often used interchangeably with "aggression" or "agonism," but it differs in its precision, and that difference is worth recognizing. While "aggression" is normally defined as any behavior relating to attack, threat, or defense, and "agonism" includes all of those behaviors plus fleeing, "violence" is more specific. 0

(C) I did a brief survey of the scientific literature over the past 4 years and found no articles that used the term "violence" when referring to animal behavior. Instead, the term was reserved for human behavior and typically for behaviors that are categorized as delinquent or antisocial. 2


① A-C-B ② B-A-C ③ B-C-A ④ C-A-B ⑤ C-B-A

 


54. 20수완 15-03

 

Less than one hundred years ago most rural households ln the United States sustained themselves by farming.


(A) While there was a well-established division of labor along gender and age lines in many farm households, there was not a well-articulated and formalized occupational structure within most rural areas. In this social and economic context, the household, the community, and the economy were tightly bound up with one another. 1

(B) While some agricultural products were sold for money on the open market, others were produced solely for household consumption or for bartering with neighbors. All family members, including husbands, wives, and children, contributed their labor to the economic maintenance and survival of the household. 0

(C) The local economy was not something that could be isolated from society. Rather the economy was embedded in the social relations of the farm household and the rural community. 2


① A-C-B ② B-A-C ③ B-C-A ④ C-A-B ⑤ C-B-A

 


55. 20수완 15-04

 

Democratic peoples — especially Americans — respond strongly to moral narratives that cleanly distinguish between the forces of good and evil.


(A) For example, most economists believe that poorly timed public thrift — austerity — can make a bad economic situation worse. But most people have a hard time understanding why it can be right for a government to spend more than it is taking in — especially if the public deficit is used to finance current consumption. While many parents grasp the rationale of going into debt to finance a college education, they are loath to cosign loans for children's fancy cars and flat-screen TVs. 2

(B) They have a harder time coming to grips with moral complexity and ambiguity. Private and public morality sometimes diverge. 0

(C) The norms of foreign policy and war are not congruent with those of domestic affairs. And the virtues of the private household do not always map neatly onto those of the public household. 1


① A-C-B ② B-A-C ③ B-C-A ④ C-A-B ⑤ C-B-A

 


56. 20수완 16-1

 

When the natural communication systems of primates are examined, no straightforward increase in complexity from monkeys to apes to humans is observed.


(A) Many researchers characterize great ape communication systems as more limited in range than those of monkeys. For example, monkeys, but not other apes, have functionally referential alarm calls, although whether monkey calls are truly referential like human language remains contested. This particular ape-monkey difference makes biological sense. 0

(B) Apes do possess gestures to initiate play, for instance, or when infants signal they wish to be carried — many of these gestures have learned elements. However, apes seemingly do not use their gestures referentially, nor do their gestures exhibit any symbolic or conventionalized features. 2

(C) Great apes are larger and stronger than monkeys, and hence are less vulnerable to predation. Apes almost certainly didn't evolve referential alarm calls because they had comparatively little to be alarmed about. Indeed, there is little that is learned at all in the vocal communication of nonhuman apes. 1


① A-C-B ② B-A-C ③ B-C-A ④ C-A-B ⑤ C-B-A

 


57. 20수완 16-01

 

In one experiment, researchers projected a picture that was completely out of focus onto a screen.


(A) Remarkably, the individuals who were the first to think they knew the identity of the picture (and write it down) were the last to correctly identify it. Why? 1

(B) The slide was then gradually brought into sharper and sharper focus. The participants were instructed to guess what the picture showed (a fire hydrant in one case), and write it down. 0

(C) Because rather than scratch through their initial opinion as the image became clearer and they were increasingly and obviously wrong, they clung to it. People who were among the last to make an identification were more successful — simply because their first choice had a better chance of being right. 2


① A-C-B ② B-A-C ③ B-C-A ④ C-A-B ⑤ C-B-A

 


58. 20수완 16-02

 

Imagine being in an art appreciation class and having to learn the styles of different artists.


(A) Traditional teaching methods would approach the works of each artist separately so as not to confuse the learners. In a fascinating study, participants were tasked with learning the styles of 12 artists based on a viewing of six sample paintings per artist. 0

(B) Now that you have learned Picasso, let's move on to Miro..."). The other half of the learners studied the paintings in a randomized fashion, with any given artist's paintings interwoven among those of the other artists. When all participants were later shown a series of new paintings (paintings by these artists that they had not seen during the learning phase) and were asked to identify the artist, the group that viewed the interwoven learning set was much better at identifying the correct artist. 2

(C) In the experiment, half the learners got the paintings in blocks as they would be presented in the typical classroom. ("Here are six paintings by Picasso. 1


① A-C-B ② B-A-C ③ B-C-A ④ C-A-B ⑤ C-B-A

 


59. 20수완 16-03

 

In addition to appropriating (and sometimes anticipating) the modern medical model for the diagnosis of crime, Arthur Conan Doyle also perfected a literary form for Sherlock Holmes that owed its origins to the medical profession as well — the case history.


(A) Medical diagnosis became based upon a principle of "differential" semiological examination of the patient's body in the form of the medical case history. 2

(B) The new discipline of pathological anatomy as a form of medical diagnosis in the period brought with it a new epistemological imperative for the physician to observe carefully every detail of the patient's environment and physical condition, and to record that information in the form of a case study that would explore the network of possible connections between the seemingly insignificant details. 0

(C) In the case history, the physician would test and retest every conceivable set of relations until the cause of the patient's illness made sense, much as Holmes would do in his explanations of cases to Watson. 1


① A-C-B ② B-A-C ③ B-C-A ④ C-A-B ⑤ C-B-A

 


60. 20수완 16-04

 

Genetic engineering followed by cloning to distribute many identical animals or plants is sometimes seen as a threat to the diversity of nature.


(A) Conversely, the renewed interest in genetics has led to a growing awareness that there are many wild plants and animals with interesting or useful genetic properties that could be used for a variety of as-yet-unknown purposes. This has led in tum to a realization that we should avoid destroying natural ecosystems because they may harbor tomorrow's drugs against cancer, malaria, or obesity. 2

(B) However, humans have been replacing diverse natural habitats with artificial monoculture for millennia. Most natural habitats in the advanced nations have already been replaced with some form of artificial environment based on mass production or repetition. 0

(C) The real threat to biodiversity is surely the need to convert ever more of our planet into production zones to feed the ever-increasing human population. The cloning and transgenic alteration of domestic animals makes little difference to the overall situation. 1


① A-C-B ② B-A-C ③ B-C-A ④ C-A-B ⑤ C-B-A

 


61. 20수완 T1-18

 

We at Bovice Children's Trust work for underprivileged children and sometimes need volunteers to help us in our project.


(A) He ensured that the children's parents understood the program that our organization was working for and thus it helped us a lot in succeeding in enrolling these children for our program. 1

(B) Besides, Chris also engaged in popularizing our organization mission that gave us the financial assistance as well as community participation by many organizations. Hence, I would like to confirm that Chris has put in about fifty hours of community service and a certificate for his service is enclosed along with this letter for your reference. 2

(C) A few months back, Chris came forward and helped us in reaching out to the underprivileged children in the Griffin Street region. 0


① A-C-B ② B-A-C ③ B-C-A ④ C-A-B ⑤ C-B-A

 


62. 20수완 T1-19

 

There were about ten people standing around a long table, dressed in black robes and chanting.


(A) He felt he was outside a window looking in. He tried to speak, but nothing came out of his mouth. He tried to run, but his legs wouldn't move. He wanted to leave, but something seemed to be holding him there. 1

(B) As he slowly started to move back, the circle of people turned and looked at him. They waved their hands for him to come to join them. He didn't want to join them; he was afraid of what they might do to him. Despite his struggle not to join them, he was pulled slowly toward them. He was no longer in control of his own body. 2

(C) David could not make out what they were saying. He only saw their mouths move in slow motion as if he was not there at all, and he felt his hands and knees trembling with fear. Then, he caught a glimpse of a table with a shadow of someone on it. He couldn't see the face. 0


① A-C-B ② B-A-C ③ B-C-A ④ C-A-B ⑤ C-B-A

 


63. 20수완 T1-20

 

If you want a friend, get a dog.


(A) In order to be seen as legitimate, the media must be seen as truthful, accurate, unbiased, and fair. 2

(B) Journalism professors and professionals have shared this humorous, colloquial saying with countless neophytes in classrooms and newsrooms because it points to a serious underpinning. The media generally and reporters in particular do not need to be loved or even to have their motives fully understood in order to carry out their obligations to inform the public. 0

(C) But, according to Stephen Klaidman and Tom Beauchamp in The Virtuous Journalist, it is essential that the public trust the press and see it as credible in its role as watchdog over governments and their agencies. Credibility is an attitude, a belief that citizens hold about whether the news media legitimately have the power to call out elected officials or others in high positions who are not playing by the rules. 1


① A-C-B ② B-A-C ③ B-C-A ④ C-A-B ⑤ C-B-A

 


64. 20수완 T1-21

 

Psychologist and author Jeremy Dean explains in relation to cultivating happiness-boosting habits that 'unfortunately there's rather a large fly in the ointment.


(A) Dean suggests that one way that we can deal with our automatic adaptation to pleasure is by varying our habits rather than repeating them in exactly the same way over and over again. This could mean, for example, making a conscious effort to respond more consciously to the question 'how are you' (rather than saying 'Fine' every time). Introducing conscious variations in some of our habits can be effective in reducing the effects of habituation. 1

(B) That fly is habituation'. Habituation means that we adapt to positive experiences more quickly than negative ones. This means that we lose the pleasure from good habits more quickly than the pain from bad ones. 0

(C) Although Dean explains that this idea stretches the formal definition of a habit which involves the same behaviour or thought in the same situation, for 'happy' habits we need an 'automatic initiation of the behaviour, but then a continuously mindful way of carrying it out. A new type of hybrid habit: a mindful habit'. 2


① A-C-B ② B-A-C ③ B-C-A ④ C-A-B ⑤ C-B-A

 


65. 20수완 T1-22

 

Social workers need to recognize that it is the client who owns the problem and therefore has the chief responsibility to resolve it.


(A) In sharp contrast, social workers seek to establish not an expert- inferior relationship but rather a relationship between equals. The expertise of the social worker does not lie in knowing or recommending what is best for the client; it lies in assisting clients to define their problems, to identify and examine alternatives for resolving the problems, to maximize their capacities and opportunities to make decisions for themselves, and to implement the decisions they make. Many students, when they first enter social work or some other helping profession, mistakenly see their role as that of "savior" or "rescuer." 2

(B) In this respect, social work differs markedly from most other professions. Most professionals, such as physicians and attorneys, advise clients about what they ought to do. 0

(C) Doctors, lawyers, and dentists are viewed as experts. Clients' decision making in such situations is generally limited to the professional's advice. 1


① A-C-B ② B-A-C ③ B-C-A ④ C-A-B ⑤ C-B-A

 


66. 20수완 T1-23

 

While memories in STM (short-term memory) are encoded acoustically, information held in LTM (long-term memory) seems to be different.


(A) When recall was delayed for 20 minutes, however, more errors were made on list items with similar meanings. This latter finding can be explained by the use of a semantic code in LTM; items were confused when they had similar meanings, so were less likely to be recalled accurately. 2

(B) Here, memories are encoded according to their meaning - that is, using a semantic code. Baddeley provided evidence for this in an experiment that required participants to learn lists of words and recall them. 0

(C) The lists contained some items that were acoustically similar (such as mad, man, cad, can) and others that were semantically similar (such as big, long, broad, high). When asked for immediate recall, participants' errors were affected by the similar-sounding words, reflecting the use of an acoustic code in STM. 1


① A-C-B ② B-A-C ③ B-C-A ④ C-A-B ⑤ C-B-A

 


67. 20수완 T1-24

 

One key to the problem of the shopping information gap is to realize that we need a better way of presenting and organizing information.


(A) Such a method is to organize information into different levels, where different amounts of consumer demand for information are presented at each level. Any shopper, at any point in the shopping experience, has a certain level of interest in products. 0

(B) If a consumer has a high level of interest then more information should be available to him. Conversely, if a consumer is indifferent to a product, then he should not be overwhelmed with information. 2

(C) Sometimes this level of interest is intense, where the consumer just cannot get enough, and sometimes this level is one of indifference, where the consumer is just browsing. The best way to convey information to consumers is to look at them as shopping on a number of possible levels of interest and to direct information to them according to their level. 1


① A-C-B ② B-A-C ③ B-C-A ④ C-A-B ⑤ C-B-A

 


68. 20수완 T1-25

 

The graph above shows the percentage of the US population with total diabetes (diagnosed and undiagnosed) and the percentage with undiagnosed diabetes both as a whole and by ethnic/racial subgroup in 2011 – 2012.


(A) Non-Hispanic Asians showed the highest proportion of undiagnosed diabetes among all of the ethnic/racial subgroups. All Hispanics showed the highest percentage of total diabetes, with 10 percent undiagnosed diabetes. 1

(B) The percentage of all Hispanics with undiagnosed diabetes was higher than that of overall Americans with undiagnosed diabetes. 2

(C) Non-Hispanic whites had the lowest percentage of total diabetes among all ethnic/racial subgroups. More than 1 in 5 non-Hispanic blacks had total diabetes, and non-Hispanic blacks had a lower proportion of undiagnosed diabetes than non-Hispanic Asians. 0


① A-C-B ② B-A-C ③ B-C-A ④ C-A-B ⑤ C-B-A

 


69. 20수완 T1-26

 

Caspar David Friedrich, a 19th-century German Romantic landscape painter, began with topographical drawings in pencil and sepia wash and did not take up oil painting until 1807.


(A) Friedrich had a severe stroke in 1835 and returned to his small sepias. He was virtually forgotten at the time of his death and his immediate influence was confined to members of his circle in Dresden, notably Georg Friedrich Kersting, who sometimes painted the figures in Friedrich's work. 1

(B) It was only at the end of the 19th century, with the rise of Symbolism, that Friedrich's greatness began to be recognized. Most of his work is still in Germany. 2

(C) His choice of subjects often broke new ground and he discovered aspects of nature so far unseen: an infinite stretch of sea or mountains, snow-covered or fog-bound plains seen in the strange light of sunrise, dusk, or moonlight. He seldom used obvious religious imagery, but his landscapes convey a sense of haunting spirituality. 0


① A-C-B ② B-A-C ③ B-C-A ④ C-A-B ⑤ C-B-A

 


70. 20수완 T1-29

 

Modern science emerged out of traditions that are clearly Western.


(A) It is important to recognize that thinking scientifically is not natural or automatic. Indeed learning to perceive the world in ways consistent with this dichotomous, observation-based perspective is something we must learn. For some of us, this may seem to be very natural. 1

(B) In actuality it is an extension of the cultural traditions within which we were raised. Classifying is an example of a very particular way of thinking that teachers must introduce to students to familiarize them with the culture of science. 2

(C) It seems that the process of sorting into either/or categories can be traced to ancient Greek thought. The process of organizing objects according to whether they have or do not have a particular property, with no intermediate category, is sometimes called Aristotelian. This very formal approach to classifying, unlike the version of classifying we might use in everyday living, makes no allowance for fuzziness. 0


① A-C-B ② B-A-C ③ B-C-A ④ C-A-B ⑤ C-B-A

 


71. 20수완 T1-30

 

Each of us views reality through the thoughts we have accumulated in our individual preserved state of awareness.


(A) Pay attention the next time you disagree with somebody. Notice how your mind instantly produces thoughts that validate your point of view. For example, as your friend defends a political candidate who you dislike, notice the rush of opposing thoughts that enter your mind. 1

(B) As this happens, consider for a moment where these thoughts came from and why you produced them at this particular moment. Consider their purpose. You did attract them and they do have a purpose. Their purpose is to reinforce your beliefs. 2

(C) When you encounter a new thought, you call upon this state to derive meaning, agree, disagree, judge, or remain neutral. Every thought you encounter or conjure up you filter through this state. You can observe this scrutiny as you produce thoughts that stimulate your feelings. 0


① A-C-B ② B-A-C ③ B-C-A ④ C-A-B ⑤ C-B-A

 


72. 20수완 T1-31

 

As an academic discipline, architecture is outside of the humanities — so in that sense there's no question that it is not one of the humanities.


(A) What I can say positively as an outsider is that architecture is a discipline seeking self-definition, and for that self-definition it looks outside of itself, to see what others say about it. 2

(B) Not institutionally at least, even if there is something of a family resemblance. Architecture students aren't oriented to thinking, reading, and writing in quite the same way as are students within the humanities. 0

(C) What's interesting about architecture is that it has always been unsure as to where to position itself and its own identity as a discipline: it is itself internally divided about whether it is a science, a technological discipline, or a mode of art or aesthetic production. This uncertainty regarding its own identity has led it to be quite open to philosophical and critical theory in a way that is unimaginable for other disciplines, like engineering or medicine, for example. 1


① A-C-B ② B-A-C ③ B-C-A ④ C-A-B ⑤ C-B-A

 


73. 20수완 T1-32

 

Important work regarding preservatives indicated that making organ meats look familiar (through their cuts, shapes, and packaging) influenced perceptions of taste.


(A) This insight was found during research on what made preserved foods most acceptable. At the beginning of World War II, there was a need for canned meats that tasted like fresh meat, for powdered milk that was reconstituted to taste like fresh whole milk, and for preserved bread that tasted like fresh bread. 0

(B) Because of this work, initial efforts introduced some organ meats as filler in ground beef and sausages. In both ground meat and sausage forms, replacing existing meat with organ meats was accepted because they did not cause the meat to look different than expected. 2

(C) The government pushed food companies to preserve foods to resemble fresh foods. Because they looked and tasted fresh, people believed they must be safe and that preservatives were not harmful. 1


① A-C-B ② B-A-C ③ B-C-A ④ C-A-B ⑤ C-B-A

 


74. 20수완 T1-33

 

Most people exhibit physical symptoms when they lie.


(A) This technology works nicely for most people. Most of us have brains that want to tell the truth, want to be trusted, and we feel guilty when we lie or when we think we will be caught. Our bodies respond to our thoughts and feelings. 1

(B) There is even new brain imaging lie detector equipment and companies springing up, such as No Lie MRI, to swear in court whether or not you are telling the truth. Not only does your body react to lies, your brain does as well. Whenever most people lie, their brain becomes overall much more active than when telling the truth. It really does take more out of you to lie than to tell the truth. 2

(C) Their bodies respond to the stress they feel when they think they will be found out. Immediately, their hearts beat faster as adrenaline bathes their system, their breathing becomes more shallow in response to feelings of panic, their hands become colder as blood flow becomes constricted, and their hands sweat and their muscles become tense. Examiners can see this pattern on the lie detector equipment and the liar is caught. 0


① A-C-B ② B-A-C ③ B-C-A ④ C-A-B ⑤ C-B-A

 


75. 20수완 T1-34

 

Social domain theory views emotions and moral judgments as reciprocal processes that cannot be disentangled.


(A) Rather, the assumption is that affective experiences are an important component of moral judgment and that the latter involves a complex integration of thoughts, feelings, and experiences. To borrow from Kant's famous saying, moral reasoning without emotion is empty; emotions without reasoning are blind. 1

(B) This view differs from emotivist or intuitionist approaches to morality, which are principally based on research with adults and give priority to emotional and implicit processes while avoiding reasoning as largely post hoc rationalizations. From the social domain perspective, this treatment of emotions and reasoning as distinct, opposing influences represents a false dichotomy. 0

(C) Children's affective experiences influence their understanding, encoding, and memory of moral violations and are part of a complex evaluative process. Information obtained from observing the affective consequences of acts for others, as well as past or immediate emotional responses to moral situations, may constitute the foundation on which moral understanding is constructed. 2


① A-C-B ② B-A-C ③ B-C-A ④ C-A-B ⑤ C-B-A

 


76. 20수완 T1-35

 

Creativity researchers now believe that creativity cannot always be defined as a property of individuals; creativity can also be a property of groups.


(A) In the same way, organization theorists and socioculturalists argue that problem solving in work groups, and learning in classrooms and informal settings, often occurs in social interactions characterized by group creativity. 1

(B) These researchers are still exploring the complex relationship between group creativity and the creativity of individual members of the group. These are different types of creativity; the interactional processes of group creativity that we can observe among improvising actors are not likely to look anything like the cognitive processes going on within any single actor's head. 2

(C) For example, the performance that is generated by an improvisational theater ensemble is the creative product of the entire ensemble; there is no way to attribute the performance to any single member of the group. 0


① A-C-B ② B-A-C ③ B-C-A ④ C-A-B ⑤ C-B-A

 


77. 20수완 T1-36

 

Socrates provided a theater of presence in Athens for the young people who gathered to watch him show that their elders — even the ones with the most brilliant reputations for wisdom — could not answer the vital questions he put to them, questions like "What is justice?"


(A) It was also dramatic, more tightly focused on the false claims to wisdom that people actually made in the public places of Athens, false claims that affected directly the young men who clustered around Socrates as his audience. And the elders who were refuted, did they gain wisdom? 1

(B) In most cases, they plainly did not. They were deep inside the space of theater, unable to see themselves, and all they knew at the end was that they had been humiliated by a fiendishly clever man. So they went away not wiser but more angry at Socrates and his cleverness. 2

(C) And so these watchers were drawn into philosophy as they learned something about human limitations. They could have picked this lesson up from comedy, but Socratic theater was more immediate, more transformative of watchers into thinkers. 0


① A-C-B ② B-A-C ③ B-C-A ④ C-A-B ⑤ C-B-A

 


78. 20수완 T1-37

 

Wildlife management in North America is complicated because most land is owned by individuals who can restrict public access or manage it as they see fit, so long as their actions do not adversely impact other people.


(A) When English colonies were established in America, colonial governments administered on the crown's behalf, and they owned the wildlife. When the colonies gained their independence following the American Revolution and became states, they continued to own wildlife but now on behalf of "the people" who had replaced the crown as the ultimate authority on which government was based. 1

(B) Wildlife, however, is not owned by individuals or landowners but by society. This is the result of English common law, which stated that wildlife was owned by the crown. 0

(C) Consequently, in North America, we have this interesting phenomenon in which society owns and manages the wildlife, but private individuals own and manage the land and habitat on which wildlife depend. Clearly a partnership is required between landowners and government if the wildlife resource is going to be managed wisely. 2


① A-C-B ② B-A-C ③ B-C-A ④ C-A-B ⑤ C-B-A

 


79. 20수완 T1-38

 

The concept of 'producer responsibility' reflects a fundamental legal principle for the allocation of responsibilities: the person acting must ensure that each action is performed in a way that does not cause damage or nuisance to other legally protected interests.


(A) It is generally accepted that these preventive responsibilities apply to manufacturing processes and include the waste generated by the manufacturing process. It seems also accepted that this principle of allocation of responsibilities also applies to manufactured goods. 1

(B) Products placed on the market must be safe and secure in general. For certain products, such as pesticides, genetically modified organisms and vehicles, the placing of these products on the market is subject to a prior licensing scheme. 2

(C) In relation to the manufacturing of goods, this concept requires operators to prevent pollution, hazards, the risk of damaging public health and damage to neighboring properties — or at least to minimize these negative externalities to a certain level defined by legislation and/or a specific permit. Hence, the preventative costs must be paid by the producer. 0


① A-C-B ② B-A-C ③ B-C-A ④ C-A-B ⑤ C-B-A

 


80. 20수완 T1-39

 

Up until 2 million years ago, the relative brain size of our hominid ancestors was the same as that of the great apes today.


(A) As our head started to increase in size to accommodate our expanding brains, this put pressure on hominid mothers to deliver their babies before their heads got too big. However, this is not a problem for our nearest non-human cousins, the chimpanzee. 1

(B) In terms of movement, chimps do not naturally walk upright and so did not develop a narrow pelvis. Their birth canals are large enough to give a relatively easier birth to their babies, which is why chimpanzees waddle when they do try to walk upright. They usually deliver by themselves in less than 30 minutes, whereas human delivery takes considerably longer and is most often assisted by other adults. 2

(C) However, something happened in our evolution to change the course of the development of our brain, which grew significantly larger. Human brain size increased to be 3-4 times larger than the brain of our ancestral apes. 0


① A-C-B ② B-A-C ③ B-C-A ④ C-A-B ⑤ C-B-A

 


81. 20수완 T1-40

 

Historians have observed that the peak age for scientific productivity was about 25 years of age in the year 1500, but by 1960 it was 37.


(A) There's been no increase in the productivity of innovators beyond middle age to make up for this shortened career, and as a result, there's been a decline in innovative output per researcher over the century. 2

(B) His data showed that the peak age increased because early age innovation is declining, and that's a result of the increased educational demands required to acquire the knowledge necessary to contribute to an innovation. At the beginning of the 20th century, great minds began their work at age 23; at the end, at age 31. 1

(C) The increasing complexity of scientific domains seems to have caused this increase; this complexity makes the ideation and elaboration rates decline, and this results in a later career peak. Educational psychologist B. F. Jones studied 700 Nobel Prize winners and technological inventors in the 20th century and found that over the course of the century, the greatest achievements occurred at later and later ages; the mean age at great achievement rose by about six years over the century, and in 2000, the peak age was 36 to 40. 0


① A-C-B ② B-A-C ③ B-C-A ④ C-A-B ⑤ C-B-A

 


82. 20수완 T1-4142

 

Environmental problems associated with agricultural production for export in developing countries can be difficult to tackle using conventional regulations because producers are typically so numerous and dispersed, while regulatory agencies are commonly inadequately funded and staffed.


(A) The authors found that organic certification did improve coffee growers' environmental performance. Specifically, they found that certification significantly reduced the use of pesticides, chemical fertilizers, and herbicides, and increased the use of organic fertilizers. In general, their results suggest that organic certification has a stronger causal effect on preventing negative practices than on encouraging positive ones. 2

(B) One study examined this question for certified organic coffee grown in Turrialba, Costa Rica, an agricultural region in the country's central valley, about 40 miles east of San Jose, the capital city. This is an interesting case because Costa Rican farmers face significant pressure from the noncertified market to lower their costs, a strategy that can have severe environmental consequences. In contrast, organic production typically not only involves higher labor costs, but the conversion from chemically based production can also reduce yields. In addition, the costs of initial certification and subsequent annual monitoring and reporting are significant. 1

(C) In principle, eco-certification of production could circumvent these problems by providing a means for the socially conscious consumer to identify environmentally superior products, thereby providing a basis for paying a price premium for them. These premiums, in turn, would create financial incentives for producers to meet the certification standards. Do socially conscious buyers care enough to actually pay a price premium that is high enough to motivate changes in the way the products are produced? Apparently, for Costa Rican coffee at least, they do. 0


① A-C-B ② B-A-C ③ B-C-A ④ C-A-B ⑤ C-B-A

 


83. 20수완 T1-4345

 

At the time, nobody in France would have accepted a woman as an apprentice, so I decided to move to Brussels for a three-year course.


(A) After returning to our Niedermorschwihr shop from Paris, I started baking and making pastry on a daily basis. Until then, my father had kept that only for weekends and celebrations. He was very pleased to accept this help because it gave him more time for his own passion to be a cook. As a baker, he had always worked like a cook, by experimenting, testing, tasting and correcting. Today, I take a similar approach with my jams. When you are creative, you cannot simply follow recipes. 1

(B) When I got back in 1978, my father had plans to expand his shop, and wanted me to work with him. However, I still had dreams of seeing the world. In the end, we found a compromise: since boys had to go away for a year for their military service, I, too, would be allowed to travel for a year. I knew that no laboratory in Paris would ever accept a woman, so I decided to enter an artistic baking competition — which I won. After this, Monsieur Peltier, who had the best store in Paris, invited me to join him for a year. He gave me a very thorough and precise training in baking and confectionery, and shared with me his avant-garde spirit. 0

(C) It is important to do what you like to do, the way you like to do it. However, I started feeling lonely in this tiny 360-inhabitant village which was too remote to attract anybody. I discussed the idea of opening a store in town with my father, on a busy street. He said, "If this were the condition for success, then how could you explain the success of Marc Haeberlin and his restaurant being awarded three Michelin stars in the tiny Illhaeusern in Alsace?" He told me to be patient, to play the long game, and to do what I love. Deep down, I knew he was right, and decided to stay and fully embrace pastry making at Niedermorschwihr. 2


① A-C-B ② B-A-C ③ B-C-A ④ C-A-B ⑤ C-B-A

 


84. 20수완 T2-18

 

Ms. Martineau and I wish to thank you for the pleasure of allowing us to interview you J last Thursday.


(A) We were delighted to learn about your superb academic record, and we also appreciated your attentiveness in listening to the description of our company's operations. However, we had many well-qualified applicants who were interested in the advertised position of human resources assistant. 0

(B) We hired a female graduate who had most of the qualities we sought. Although we realize that the job market is difficult at this time, you have our heartfelt wishes for good luck in finding what you are looking for. 2

(C) As you may have guessed, we were particularly eager to find a minority individual who could help us fill out our employment equity goals. Although you did not fit one of our goal areas, we enjoyed talking with you. 1


① A-C-B ② B-A-C ③ B-C-A ④ C-A-B ⑤ C-B-A

 


85. 20수완 T2-19

 

I watched with curiosity as a strange couple walked through the house taking measurements.


(A) It was a small one, but very poisonous. Our servant, Niti, was called in to dispose of it, but he was also at a loss for what to do. 2

(B) When I was at the threshold of the living room, Mother, seeming quite anxious, shouted, "Stay away, Pieter, stay away!" I soon learned what all the fuss was about. In their enthusiasm, the decorators had moved a few cartons of books that were stored in a corner and managed to disturb a snake, which slithered out and frightened them. 1

(C) The process took about two hours, so I lost interest and hurried back to my own project outside. I had nearly forgotten about our visitors when I heard a sudden scream. I raced back into the house. 0


① A-C-B ② B-A-C ③ B-C-A ④ C-A-B ⑤ C-B-A

 


86. 20수완 T2-20

 

Overconfidence about their social media skills prevents most social media managers from getting the necessary training.


(A) Be skeptical of the self-proclaimed social media mavens, rock stars, or experts who have no practical experience running, maintaining, and building online communities around causes or nonprofit brands on social networking sites. 2

(B) Take Webinars, attend boot camps, or go to conferences that focus on the how to of social media. The most valuable advice should be practical and should come from someone who actually manages online communities and works with these tools every single day. 1

(C) Every second of every day, nonprofits on various kinds of social media are making obvious mistakes that are counterproductive to their presence on these sites. Please get training! 0


① A-C-B ② B-A-C ③ B-C-A ④ C-A-B ⑤ C-B-A

 


87. 20수완 T2-21

 

After his failure in Edinburgh, Darwin went to the University of Cambridge to study for a basic arts degree, with the idea that he would become a clergyman.


(A) They inspired him to become a naturalist. John Henslow took him plant collecting in the Cambridge countryside. Adam Sedgwick went with him to Wales to study the local rocks and fossils. 1

(B) He passed his exams. Just. But Cambridge turned out to be vitally important because of the friendships he forged with the professors of botany and geology. 0

(C) After this tour with Sedgwick, Darwin had graduated from the university and was at a loose end, not sure what to do next. He was saved by an unusual offer: would he like to become the 'gentleman naturalist' on a surveying voyage aboard the ship HMS Beagle, led by Captain Robert Fitzroy of the Royal Navy? His father said no, but his uncle convinced him that it was actually a great idea for his son. The voyage on the Beagle was the making of Charles Darwin. 2


① A-C-B ② B-A-C ③ B-C-A ④ C-A-B ⑤ C-B-A

 


88. 20수완 T2-22

 

To attract the best people and succeed as a business, the authentic organization of the future will need to foster environments where creativity and innovation are at a premium, employees feel engaged and committed, and leadership pipelines are carefully cultivated for future success.


(A) Case in point: Back in the 1980s, the business division of a US publisher had one of the most widely diverse workforces. One senior editor had been part of a Washington think tank and was an expert on Asian culture; another held a PhD in American history; another had worked as a speechwriter for a US president and was an environmental activist. 1

(B) In our research, workplaces with those qualities look for an unusual kind of diversity, hiring people for differences that are more than skin deep. Differences in thought processes, frames of references and skills, among other things. 0

(C) There was also an associate editor who had interned at the New Yorker magazine and another who had a background in foreign affairs. Only two of those editors held MBAs — and this was the business division! 2


① A-C-B ② B-A-C ③ B-C-A ④ C-A-B ⑤ C-B-A

 


89. 20수완 T2-23

 

The pursuit of flavor is one path to a good life, a truth recently discovered by Americans that demands a new account of the meaning of food and its consumption.


(A) "Pursue pleasure only in moderation," say countless sages throughout history; "it's only transitory," according to the timeless. "It is inimical to spirituality," the bodiless would have us believe; "it will not lead to happiness," the ubiquitous self-help books tell us. 1

(B) Any discussion of food and its place in our lives must begin with the role of pleasure. Yet our attitude toward pleasure is ambivalent: "Love People, Not Pleasure," blares the New York Times; "There Is More to Life Than Being Happy," proclaims the Atlantic Monthly. 0

(C) We spend much time and many resources pursuing pleasure but then condemn it with a fervor usually reserved for death and taxes. 2


① A-C-B ② B-A-C ③ B-C-A ④ C-A-B ⑤ C-B-A

 


90. 20수완 T2-24

 

Mobile crowd sensing for smart cities can support efficient, safe and green mobility in urban environments.


(A) Environmental sensors may also report air or noise pollution levels. This enables tourists in unfamiliar places to make even better decisions than well informed locals might take. 2

(B) Here crowd sourcing can inform people about whether to seek alternative routes, when best to arrive at attractions or restaurants, how to avoid unpleasant surprises when traveling, where to park, and which public transport solution would be best. 1

(C) Given the ubiquity of mobile devices carried by people worldwide, social mobile crowd sensing through the IoT can allow tourists to know about popular events in a destination, provide interactive feedback with other tourists at different locations, reveal the best places to be at a certain time, local weather forecasts, and expected travel times throughout the day. 0


① A-C-B ② B-A-C ③ B-C-A ④ C-A-B ⑤ C-B-A

 


91. 20수완 T2-25

 

The graph above shows pet ownership in the US, with the total number of pets owned and the number of households that owned each type of pet in 2016.


(A) Regarding the total number of pets, freshwater fish were the most owned pets, whereas cats were the second most owned pets, at 139.3 million and 94.2 million pets respectively. The number of households that owned dogs was larger than the combined number of households that owned cats and households that owned freshwater fish. 0

(B) Lastly, among the six types of pets, horses were the least owned. 2

(C) Although the number of households with saltwater fish as pets was the smallest, as many as 18.8 million saltwater fish were owned. Households that owned birds ranked fourth among pet-owning households, with 7.9 million households. 1


① A-C-B ② B-A-C ③ B-C-A ④ C-A-B ⑤ C-B-A

 


92. 20수완 T2-26

 

The fame of French director Andre Cayatte reached its peak in the 1950s.


(A) As he had no valid papers, he was forced to work for Continental Films, the German production company in Paris, for which he made his first films from 1942 onwards. This was to cause him a certain amount of trouble after the liberation of Paris. 1

(B) Cayatte, a journalist, lawyer and novelist, became involved in cinema at the end of the 1930s by writing screenplays. He was taken prisoner in 1940, but managed to escape. 0

(C) After the Second World War, he gained the public's and the critics' attention through his committed films on issues of public concern, pleas against the death penalty, satires directed at the failings of the justice system, well-argued attacks against the weaknesses of family upbringing, and so on. He was strongly criticised by the film-makers of the New Wave, who accused him of concentrating too much on expounding an argument to the detriment of the aesthetic considerations of direction. He gradually withdrew from film-making in the 1970s. 2


① A-C-B ② B-A-C ③ B-C-A ④ C-A-B ⑤ C-B-A

 


93. 20수완 T2-29

 

In some ways, food media takes readers and viewers into the back of the house where gourmet cooking takes place.


(A) This is not to say such depictions are accurate, entertaining as they might be. As Andrew Chan notes, contemporary shows "create a gap that separates the viewer from the reality of actual cookery." 0

(B) As such, the media has made visible some of the hidden work that goes into producing our eating experiences but often in ways that are more fantasy than reality. 2

(C) In fact, Kathleen Collins explains that, "Television cooking shows, and the media in general, are largely responsible for glamorizing what will always be, in actuality, toiling, sweaty labor." She also concedes, however, that such media coverage — and the public's seemingly insatiable appetite for programming related to food and eating — "has brought deserved attention to a previously uncelebrated class of laborers and artisans." 1


① A-C-B ② B-A-C ③ B-C-A ④ C-A-B ⑤ C-B-A

 


94. 20수완 T2-30

 

DNA reproduces and replaces itself continually.


(A) A third-generation copy is slightly less sharp than the original, the fourth-generation copy even less so, and so forth. Eventually, the process breaks down; it is no longer possible to make a legible copy, and the message is lost. When this happens to DNA, the result is the death of a cell. 2

(B) Under optimal conditions, this reproduction is flawless and every copy is an exact replica of the one it replaces. This is normally true in children. As we age, however, we are increasingly vulnerable to factors that damage our DNA during replication. 0

(C) The body's process of DNA replication is similar to that of a copy machine making photocopies of an original, with each subsequent copy made from the previous photocopy. If the copy machine is in perfect condition, the first copy will hardly be distinguishable from the original. Gradually, though, even copies made on a good-quality machine will lose definition. 1


① A-C-B ② B-A-C ③ B-C-A ④ C-A-B ⑤ C-B-A

 


95. 20수완 T2-31

 

Sometimes, of course, we might have good reasons, conceptually and critically, to isolate the content of a film from its form.


(A) But limited access to some specific details, and the demands of cinematic storytelling, compelled the filmmakers to take liberties with the original content when giving the movie its ultimate form. Zero Dark Thirty invents and combines characters, rearranges and condenses action, and speculates on events. 1

(B) The subject of Kathryn Bigelow's Zero Dark Thirty is the tracking and eventual killing of the terrorist Osama Bin Laden by United States CIA agents and US Navy Seals. That content is historical fact. 0

(C) Many critiques of Zero Dark Thirty, noting the significance and sensitivity of the content, questioned the film's completeness, accuracy, and reliability. Yet the movie could be considered a formal success; it received Academy Award nominations for Best Picture, Actress, Original Screenplay, Film Editing, and Sound Editing. By focusing solely on content, we may risk overlooking the aspects that make movies interesting as individual works of art. 2


① A-C-B ② B-A-C ③ B-C-A ④ C-A-B ⑤ C-B-A

 


96. 20수완 T2-32

 

Even before Socrates the Greeks had come to a critical differentiation between natural and human phenomena.


(A) The crucial point of this distinction was that while nature, physis, provided the fixed context to which human beings could do nothing else but adjust their lives, human-made things and institutions, nomoi (plural of nomos), could, at least in principle, be changed by human activity. 2

(B) As early as the seventh or eighth century BC — as evidenced by the Iliad and the Odyssey —it had become customary to distinguish those objects in the surrounding world that appeared to remain always and everywhere the same — plants, animals, the seasons, the stars, and so forth — from others that seemed to vary from time to time and place to place, such as languages, customs, laws, political institutions, and so on. 0

(C) The Greeks came to refer to the former sorts of things collectively as physis, or what we would roughly translate as nature, and to the latter as nomos, or, approximately, custom or perhaps even culture. 1


① A-C-B ② B-A-C ③ B-C-A ④ C-A-B ⑤ C-B-A

 


97. 20수완 T2-33

 

The transition to a network society has, by all accounts, turned research on youth and media on its head — leading to new opportunities and new challenges.


(A) With the rapid and often short-lived developments in the media landscape, such research is far more complicated than it used to be. 2

(B) Perhaps most obvious is the fact that our object of study has become a moving target. Many of the media and communication technologies that we investigate today are continually changing — often while we try to understand the phenomenon in question. 0

(C) This is especially problematic for the social sciences, because social scientists often need sufficient time to answer questions with appropriate methodological rigor. To establish causal relationships between media use and longer-term outcomes, youth must be followed over several years, and their media use needs to be measured repeatedly. 1


① A-C-B ② B-A-C ③ B-C-A ④ C-A-B ⑤ C-B-A

 


98. 20수완 T2-34

 

Some species have evolved not only a theory of mind but also, distinctly, a theory of relationships — which is evolutionarily advantageous, because recognizing relationships between other individuals helps predict their social behavior.


(A) The most basic type of such knowledge is when one animal knows the relative dominance rank of two other animals, not just its own rank with respect to the others. This important ability is widespread, seen in hyenas, lions, horses, dolphins, and, of course, primates, but also in fish and birds. 0

(B) All three animals understand what it means for two of them to have a special bond. 2

(C) Capuchin monkeys in conflict preferentially seek out allies that they know to be higher ranked than their opponents, and they also seek out allies that they know have closer relationships with themselves than with their opponents. If two chimpanzees have a fight and a bystander offers consolation to the loser, this can reconcile the two combatants, but only if the bystander has a friendship with the aggressor. 1


① A-C-B ② B-A-C ③ B-C-A ④ C-A-B ⑤ C-B-A

 


99. 20수완 T2-35

 

Social systems exist to mitigate risk: that is why human beings live in communities in the first place.


(A) A society in which people consider themselves to be one big family is more likely to show resilience in a disaster or prolonged stress than one in which each individual models his or her behavior on self-reliant individualism, all other things being equal. This is the fundamental idea behind insurance, extended family networks of assistance, and disaster assistance. 1

(B) Therefore this class of problem often depends importantly on the level of resilience a society demonstrates under stress. 2

(C) People living in groups can protect themselves better from enemies or predators, share resources, and help one another beyond the immediate kinship group. Through technology, trade, and political organization, organized communities under stress can access help that will offset the local impact of ecological problems and diffuse the impact over a much larger area or population. 0


① A-C-B ② B-A-C ③ B-C-A ④ C-A-B ⑤ C-B-A

 


100. 20수완 T2-36

 

Daniel Merton Wegner was the first to analyze transactive memory, especially as it occurs in couples.


(A) Wegner was especially interested in the use of people as memory aids. He speculated that a transactive memory system may develop in many groups to ensure that important information is remembered. 1

(B) He noted that people often supplement their own memories, which are limited and can be unreliable, with various external aids. These include objects (eg, address or appointment books) and other people (eg, friends or coworkers). 0

(C) This system combines the knowledge possessed by individual group members with a shared awareness of who knows what. So when group members need information, but cannot remember it themselves or doubt that their memories are accurate, they can tum to each other for help. A transactive memory system can thus provide a group's members with more and better information than any of them could remember alone. 2


① A-C-B ② B-A-C ③ B-C-A ④ C-A-B ⑤ C-B-A

 


101. 20수완 T2-37

 

The manner in which people communicate has some obvious relationship to the way in which their society is organized and governed.


(A) In ancient oral societies, people were ruled by unpredictable gods who worked in mysterious ways. When human beings learned how to cast their messages and conversations into written language, they were able to begin to organize their lives in more rational systems. 0

(B) This posed a severe challenge to those among them who were priests and holy men and women; obviously these leaders regarded writing as subversive, destructive, and sinful. It didn't take them long to realize that if they could control the writing, they could regain their power as emissaries of the gods. 2

(C) They could record history and discover patterns in the weather, their food sources, and the ways of their enemies. They could take more control of their lives, solve some of nature's mysteries, and become less dependent on the whims of their gods. 1


① A-C-B ② B-A-C ③ B-C-A ④ C-A-B ⑤ C-B-A

 


102. 20수완 T2-38

 

The hypothalamus is the part of the brain that receives messages from nerves about the internal condition of the body and the external environment.


(A) For example, cells may resist the signal of the hormone insulin, causing the pancreas to overwork, secreting ever more insulin. An excess of insulin causes a series of effects on other hormones, and some become elevated while others become depressed. Many hormone imbalances can be corrected nutritionally if they are detected early enough, but some may require medication. 2

(B) It responds to this information by sending signals to the appropriate glands to release hormones. Once hormones are released into the bloodstream they travel to particular target cells to deliver their chemical messages and cause a specific biological response. 0

(C) Ideally, our hormones exist in a delicate balance and deliver their messages to the cells accurately and reliably. But hormone imbalances become increasingly common with advancing age. 1


① A-C-B ② B-A-C ③ B-C-A ④ C-A-B ⑤ C-B-A

 


103. 20수완 T2-39

 

The primary perceptual faculty in human beings, as in all primates, is vision.


(A) Until the 1970s, Zeki points out, neurological models of perception were heavily influenced by the mistaken philosophic view, probably traceable to Kant, that "sensing" reality and "understanding" (grasping) it are fundamentally disparate phenomena. Now, however, through sophisticated techniques for studying both normal subjects and patients who have suffered various impairments of brain function, we know that normal perception entails simultaneous "seeing" and "understanding." 1

(B) Specialized areas of the brain not only detect visible attributes such as color, form, and motion but also "identify" and integrate them into a unified, coherent "picture." The integration of visual information that results constitutes both sensory perception and recognition of the visual world. 2

(C) Much of the improved understanding of perceptual processes has therefore derived from the neurobiology of visual perception. As Semir Zeki, a principal researcher in the field, has observed, the study of vision is a "profoundly philosophical enterprise," for it constitutes an inquiry into "how the brain acquires knowledge of the external world, which is no simple matter." 0


① A-C-B ② B-A-C ③ B-C-A ④ C-A-B ⑤ C-B-A

 


104. 20수완 T2-40

 

Choosing similar friends can have a rationale.


(A) However, the carrying capacity of a given environment places a limit on this strategy. If resources are very limited, the individuals who live in a particular place cannot all do the exact same thing (for example, if there are few trees, people cannot all live in tree houses, or if mangoes are in short supply, people cannot all live solely on a diet of mangoes). 1

(B) Assessing the survivability of an environment can be risky (if an environment turns out to be deadly, for instance, it might be too late by the time you found out), so humans have evolved the desire to associate with similar individuals as a way to perform this function efficiently. This is especially useful to a species that lives in so many different sorts of environments. 0

(C) A rational strategy would therefore sometimes be to avoid similar members of one's species. 2


① A-C-B ② B-A-C ③ B-C-A ④ C-A-B ⑤ C-B-A

 


105. 20수완 T2-4142

 

Talents, unlike strengths, are largely innate.


(A) For example, it is possible to train oneself to be diligent, patient, and kind. Strengths can be acquired by almost any individual as long as there is time, effort and commitment. Strengths are therefore usually far more voluntary; there are many choices when it comes to strengths. You can decide whether you want to have a particular strength, whether you want to use it and develop a strength even further, whether to keep building it and to some extent whether to acquire it in the beginning. Also, strengths cannot be squandered. 2

(B) Of course they involve some choices, but the choices are in the context of choosing whether to use and develop the talent rather than about possessing that talent in the beginning. Talents are abilities that are brought to the surface when we begin to use them when pursuing various activities. Hence the importance of encouraging young children to engage in a variety of experiences so that they have opportunities to discover their talents. For example, some very young children naturally have good hand-eye coordination which makes them very good at being able to do things such as hitting a ball. 0

(C) This talent can be enhanced to the point of exceptional ability. Malcolm Gladwell suggests, however, that this requires ten thousand hours of practice before the age of 18 years. Strengths, on the other hand, are personality traits that can be acquired through active intentions such as practising, reading about them or learning from others. Just because we were not born with certain personality traits doesn't mean we can't develop them. 1


① A-C-B ② B-A-C ③ B-C-A ④ C-A-B ⑤ C-B-A

 


106. 20수완 T2-4345

 

President Carter used the power of emotions during the historic peace negotiations between Israel and Egypt.


(A) This groundwork did not just "happen." It took honest work. Carter and Begin began to establish rapport at their first meeting more than a year prior to the negotiation. They met at the White House, where Carter invited the Prime Minister for an open, private discussion about the Mideast conflict. Months later, Carter and his wife invited Begin and his wife to a private dinner, where they talked about their personal lives. Later, during the Camp David negotiation, Carter demonstrated that he was looking out for each party's welfare. For example, before Begin met with Sadat for the first time at Camp David, Carter alerted Begin that Sadat would present an aggressive proposal; he cautioned Begin not to overreact. Carter did not want the negotiation to fail, nor did Begin or Sadat. Positive emotions between Carter and each leader helped to move the negotiation forward. 2

(B) He invited Israel's Prime Minister, Menachim Begin, and Egypt's President, Anwar Sadat, to Camp David. His goal was to help the two leaders negotiate a peace agreement. After thirteen long days, the negotiation process was breaking down. The Israelis saw little prospect for reaching agreement. By this time, Carter had invested a lot of time and energy in the peace process. He could easily have expressed frustration, perhaps approaching Begin with a warning to accept his latest proposal "or else." But an adversarial approach might have caused Begin to abandon the negotiation process completely. Instead, Carter made a gesture that had a significant emotional impact. Begin had asked for autographed pictures of Carter, Sadat, and himself to give to his grandchildren. 0

(C) Carter personalized each picture with the name of a Begin grandchild and handed Begin the photographs. Begin saw his granddaughter's name on the top photograph and spoke her name aloud. His lips trembled. He shuffled through the photographs and said each grandchild's name. He and Carter talked quietly about grandchildren and about war. This was a turning point in the negotiation. Later that day, Begin, Sadat, and Carter signed the Camp David Accord. The open discussion between Carter and Begin could not have happened if there were a poor relationship between them. The groundwork of positive emotions allowed nonthreatening conversation about serious differences. 1


① A-C-B ② B-A-C ③ B-C-A ④ C-A-B ⑤ C-B-A

 


107. 20수완 T3-18

 

Mr Liam Baker, wants to return a pair of shoes he bought from this showroom a week ago.


(A) Mr Newman and I tried to convince him about our policies and explained worn shoes are not refundable, but it was all in vain. Finally, to prevent him from creating a scene, we promised that we would let him know our decision after consulting the head office. 1

(B) He claims that there is a manufacturing defect in the design. In fact, he is accusing us of selling seconds from this shop. 0

(C) I have also been successful in convincing him to take the shoes back with him. But he will certainly want an answer from us within a couple of days. Please advise us as to what course of action we should take. 2


① A-C-B ② B-A-C ③ B-C-A ④ C-A-B ⑤ C-B-A

 


108. 20수완 T3-19

 

Jake lifted his head high and tried to control his breathing.


(A) She said it with such conviction that Jake's heart felt as if it would explode. Jake then repeated the same vows. He placed the ring upon Leah's finger. Someone cleared their throat. "You may now kiss your bride." 2

(B) Leah smiled at him, and his heart beat double time. He still had a hard time believing that Leah loved him and would soon be his wife. He only half listened to the wedding vows. Leah's voice was soft when she vowed to "love and to cherish, until death do us part." 1

(C) He would start his new life with Leah. His heart pounded in his chest with joy. Jake watched as a vision of beauty floated down the stairs. He'd never seen anyone look as lovely in his whole life. 0


① A-C-B ② B-A-C ③ B-C-A ④ C-A-B ⑤ C-B-A

 


109. 20수완 T3-20

 

Most of the eating people do while standing up is impulse eating, like nibbling on something you see when opening the fridge to get something else out, grabbing a chocolate as you walk past an open box sitting on your colleague's desk or taking little bites as you prepare food or clean up after meals.


(A) But here's the thing: all the little nibbles here and there do count and will have consequences. 1

(B) It's important that you make every bite count. Not only will eating sitting down make you feel more physically satisfied, it will also make you feel more psychologically satisfied if you see a full meal or snack spread out in front of you rather than grabbing bites here or there while on the move. 2

(C) There's a real tendency to think that little nibbles here or there while going about your business don't actually count or won't have any consequences. 0


① A-C-B ② B-A-C ③ B-C-A ④ C-A-B ⑤ C-B-A

 


110. 20수완 T3-21

 

The objective of battle, to "throw" the enemy and to make him defenseless, may temporarily blind commanders and even strategists to the larger purpose of war.


(A) And one side's will has to be transmitted to the adversary at some point during the confrontation (it does not have to be publicly communicated). A violent act and its larger political intention must also be attributed to one side at some point during the confrontation. History does not know of acts of war without eventual attribution. 2

(B) This insight was famously captured by Clausewitz's most famous phrase, "War is a mere continuation of politics by other means." To be political, a political entity or a representative of a political entity, whatever its constitutional form, has to have an intention, a will. That intention has to be articulated. 1

(C) War is never an isolated act, nor is it ever only one decision. In the real world, war's larger purpose is always a political purpose. It transcends the use of force. 0


① A-C-B ② B-A-C ③ B-C-A ④ C-A-B ⑤ C-B-A

 


111. 20수완 T3-22

 

In the field of education, the sequence of human ages is reflected in the relationship between teacher and student.


(A) What needs to be remembered, however, is that classics is not simply a synonym for oldness or for wisdom construed in the past tense. Treated in this manner, the classics would be a pastime reserved only for teachers and older people, which would vitiate the need for continuity of transmission. 2

(B) A short label for these ripened fruits of learning is the "classics," and in this sense, every major cultural tradition on earth can boast a storehouse of classical texts and insights. 1

(C) In past centuries, this relationship was a close and intimate bond of apprenticeship (paradigmatically captured in the Indian formula of guru-shishyaparampara) in which the teacher transmitted to the student not only information but also the continuity of a tradition of learning, the fruits of the slow labor of intellectual and moral seasoning (far removed from clever dexterity). 0


① A-C-B ② B-A-C ③ B-C-A ④ C-A-B ⑤ C-B-A

 


112. 20수완 T3-23

 

The words used in survey questions should not trigger biases, unless doing so is the researcher's conscious intent.


(A) Take the difference between welfare and assistance for the poor. On average, surveys have found that public support for more assistance for the poor is about 39 percentage points higher than for welfare. Most people favor helping the poor. 1

(B) Biased words and phrases tend to produce misleading answers. Some polls ask obviously loaded questions, such as "Isn't it time for Americans to stand up for morality and stop the shameless degradation of the airwaves?" Especially when describing abstract ideas (eg, freedom, justice, fairness), your choice of words can dramatically affect how respondents answer. 0

(C) Most people oppose welfare. The "truly needy" gain our sympathy, but "loafers and bums" do not. 2


① A-C-B ② B-A-C ③ B-C-A ④ C-A-B ⑤ C-B-A

 


113. 20수완 T3-24

 

Competitive activities can be more than just performance showcases where the best is recognized and the rest are overlooked.


(A) Performance feedback requires that the program go beyond the "win, place, or show" level of feedback. Information about performance can be very helpful, not only to the participant who does not win or place but also to those who do. 2

(B) The provision of that type of feedback can be interpreted as shifting the emphasis to demonstrating superior performance but not necessarily excellence. The best competitions promote excellence, not just winning or "beating" others. The emphasis on superiority is what we typically see as fostering a detrimental effect of competition. 1

(C) The provision of timely, constructive feedback to participants on performance is an asset that some competitions and contests offer. In a sense, all competitions give feedback. For many, this is restricted to information about whether the participant is an award- or prizewinner. 0


① A-C-B ② B-A-C ③ B-C-A ④ C-A-B ⑤ C-B-A

 


114. 20수완 T3-25

 

The two pie charts above show past world energy consumption for 2011 and projected world energy consumption for 2040.


(A) "Biomass" accounted for 4 percent of world energy consumption for 2011, ranking fifth. By 2040, the consumption of "Other Renewables" and "Nuclear" is expected to increase by 1 percentage point, respectively. 1

(B) In 2011, the combined share of "Petroleum", "Natural Gas", and "Coal" accounted for more than three quarters of world energy consumption, and the share of each fuel is expected to decrease except for "Natural Gas" in 2040. "Petroleum" is expected to remain the most consumed fuel, even with its share projected to fall from 3 7 percent in 2011 to 31 percent in 2040. 0

(C) "Hydropower" was the second least consumed energy source at 3 percent in 2011 and is expected to account for the same share in 2040. 2


① A-C-B ② B-A-C ③ B-C-A ④ C-A-B ⑤ C-B-A

 


115. 20수완 T3-26

 

The eagle dance is a dramatization of the relationship believed to exist between man and the eagle and deific powers.


(A) On the head, is a close fitting headdress covered with feathers; the eyes are indicated, and at the front, is a long, curved beak —in all, a very good representation of an eagle's head. Over the shoulders and attached to the arms, are great feathered wings, and a feathered tail is attached to the belt in the back. This dance is a favorite with the public and is frequently performed at public exhibitions. 2

(B) Although the costume may vary from pueblo to pueblo, the basic features are the same. Each dancer's body is painted realistically; he wears a kilt, usually decorated with an undulating snake design. 1

(C) Two young men are costumed as eagles, one a male and the other female; in the course of the dance, they imitate almost every movement of these great birds. One sees them in the act of soaring, hovering over the fields, circling, perching on high places, and resting on the ground. 0


① A-C-B ② B-A-C ③ B-C-A ④ C-A-B ⑤ C-B-A

 


116. 20수완 T3-27

 

Free Coffee Event This is an Awesome Gift of Coffee for Coffee Lovers by Kaffa Kaldi.


(A) But you can only redeem one coffee from each outlet per week. Are there minimum age requirements to enter the event? - There is no minimum age to redeem a free coffee. But we do not encourage children below the age of 16 to drink coffee. 2

(B) FAQs : How can this event last for a full year? - The sponsors for this agree to extend this event for the whole of 2020, so you can enjoy a free coffee on a weekly basis. So, how many free coffees can I redeem per week? - You can redeem a free coffee from any of the outlets on the list. 1

(C) Where can I get a free coffee? Sign up today and we will send you information on where and how to get your free coffee. There is no purchase required (though some very good and reasonably priced food is being offered for all participants of this program). Simply show your vouchers to redeem your free coffee! 0


① A-C-B ② B-A-C ③ B-C-A ④ C-A-B ⑤ C-B-A

 


117. 20수완 T3-28

 

Fun Run and Walk Saturday, October 10, 2020 Red South Beach, San Diego, CA Join Barnes Community Clinic (BCC) for our first Fun Run and Walk event!


(A) Register by September 30 to ensure your bib has your name on it. • Event Details - 3 Mile Run: 9 a.m. - 1 Mile Walk: 9:30 a.m. - Barnes Community Festival: 11 a.m. The festival is held in downtown Barnes, featuring sidewalk sales and family entertainment from a wide variety of local vendors. 2

(B) • Register Early and Save - Online registration: $20 until September 15, $25 until September 30 - Race day registration: $30 (opens 7 a.m.) • Amenities The first 500 participants who register online get a free T-shirt. Plus, everyone gets a beautiful custom bib. 1

(C) This is a fun noncompetitive event, and open to everyone. The best part is that 100% of the registration fees go back directly to BCC, helping to give patients the best medical care possible. 0


① A-C-B ② B-A-C ③ B-C-A ④ C-A-B ⑤ C-B-A

 


118. 20수완 T3-29

 

In graphic form and function, the newspaper is coming to resemble a computer screen, as the combination of text, images, and icons turns the newspaper page into a static snapshot of a World Wide Web page.


(A) USA Today in fact makes considerable use of "hypertextual" links back and forth throughout its pages, and these links are sometimes cued by small graphics. The purpose of these icons together with the other pictures and graphics is not merely decorative. 1

(B) Together they help to redefine the function of the newspaper, which is no longer only to transmit verbal information, but also to provide an appropriate visual experience and through that experience dictate an appropriate reaction to the stories being told. 2

(C) In many newspapers the index now consists of summaries gathered in a column running down the left-hand side of the page, and a small picture is often included with the summary. Anyone familiar with multimedia presentations can easily read such a picture as an iconic button, which the user would press in order to receive the rest of the story. 0


① A-C-B ② B-A-C ③ B-C-A ④ C-A-B ⑤ C-B-A

 


119. 20수완 T3-30

 

"What's in a name?


(A) What a work of art is titled, on the other hand, has a significant effect on the aesthetic face it presents and on the qualities we correctly perceive in it. A painting of a rose, by a name other than the one it has, might very well smell different, aesthetically speaking. The painting titled Rose of Summer and an indiscernible painting titled Vermillion Womanhood are physically, but also semantically and aesthetically, distinct objects of art. 2

(B) That which we call a rose, by any other name would smell as sweet." This thought of Shakespeare's points up a difference between roses and, say, paintings. Natural objects, such as roses, are not interpreted. 0

(C) They are not taken as vehicles of meanings and messages. They belong to no tradition, strictly speaking have no style, and are not understood within a framework of culture and convention. Rather, they are sensed and savored relatively directly, without intellectual mediation, and so what they are called, either individually or collectively, has little bearing on our experience of them.1


① A-C-B ② B-A-C ③ B-C-A ④ C-A-B ⑤ C-B-A

 


120. 20수완 T3-31

 

There is a long-standing debate within psychology as to whether emotions per se are stored in memory.


(A) Remembering the circumstances in which an emotion was experienced also may cause people to experience a similar but new emotion in the present, and it is this new emotion that is then reported. As William James put it, "The revivability in memory of the emotions, like that of all the feelings of the lower senses, is very small.... We can produce, not remembrances of the old grief or rapture, but new griefs and raptures, by summoning up a lively thought of their exciting cause." 2

(B) According to this view, when asked to remember emotions, people retrieve not the fleeting emotional experience but a redescription of it based on memory for relevant details concerning the event or based on beliefs about how one is likely to have felt. 1

(C) Some investigators argue that emotion cannot be stored in memory but must be reconstructed based on knowledge concerning the circumstances in which the emotion was experienced. 0


① A-C-B ② B-A-C ③ B-C-A ④ C-A-B ⑤ C-B-A

 


121. 20수완 T3-32

 

The apparent "mess" of the bacterial domain is understandable.


(A) From the examples of the macrobiological world, it is clear that lifestyle or morphology is only of limited use to establish relatedness, and many bacteria look more or less the same under a microscope. So how should we group bacteria, if not by their looks and behavior? 0

(B) For example, shigellosis is a type of severe diarrhea caused by Shigella species, for instance Shigella dysenteriae, which, by objective criteria, are just particular nasty brands of E. coli (the "E." of E. coli stands for the genus Escherichia). There is no scientific reason to grant Shigella bacteria their own genus name, but taxonomists have not renamed Shigella bacteria to be incorporated into the Escherichia genus — yet. 2

(C) In the old days, when research was dedicated to medical microbiology, distinctions were frequently made based on the diseases bacteria could cause. This has led to some inaccurate classifications that we live with even today. 1


① A-C-B ② B-A-C ③ B-C-A ④ C-A-B ⑤ C-B-A

 


122. 20수완 T3-33

 

Many people, perhaps even you, associate philosophy with an image of bearded men in togas discussing deep thoughts.


(A) For example, in history, it is absolutely necessary to know specific facts about events. You can't study history just by reasoning or by thinking about ideas. Likewise, you can't do psychology without having a certain scientific view toward human nature and observing actual patterns in human behavior (this is why psychology is, in fact, a relatively recently developed discipline). 2

(B) There were such people in fact, and our Western intellectual heritage owes a lot to the ancient Greeks (which is why we have this mental picture). But it wasn't just the ancient Greeks who did philosophy. Philosophy is a basic human intellectual activity that has been pursued in every advanced culture since time immemorial. 0

(C) Its universality and timeless appeal should tell you something about it right off the bat. The doing of philosophy does not depend on any particular body of outside knowledge. This distinguishes it from any of the other "social sciences." 1


① A-C-B ② B-A-C ③ B-C-A ④ C-A-B ⑤ C-B-A

 


123. 20수완 T3-34

 

Musical judgments are never made in complete isolation.


(A) The formation of "taste cultures" has always been socially defined. Participation in certain genres of music was historically determined by a person's social position, not by a purely independent aesthetic choice. Indeed, from a sociological perspective, taste is always a social category rather than an aesthetic one; it refers to the way we use cultural judgments as social "currency," to mark our social positions. 0

(B) These factors contribute to a sense of the relativity of any single position. Contemporary musical choices are plural as never before, and the effect of that plurality is inevitably to confirm that, in matters of musical judgment, the individual can be the only authority. 2

(C) This may be less clear today, since contemporary society is characterized by the fragmentation of older taste cultures and the proliferation of new ones. In this context, cultural transactions take place with increasing rapidity — hence the heating up of the cultural economy and its rapid turnover of new products. Not only are taste cultures themselves shifting, but people now tend to move between them with greater ease. 1


① A-C-B ② B-A-C ③ B-C-A ④ C-A-B ⑤ C-B-A

 


124. 20수완 T3-35

 

Mathematics and logic are closely related, and in many branches of science forms of mathematics are available which are suitable for the deduction of the consequences of hypotheses.


(A) Nevertheless a sacrifice is usually made when reliance is placed on mathematics, because the existing forms of mathematics are adequate only for simplified cases. 1

(B) For example, much of organic chemistry has been developed with little assistance from mathematics. As a consequence the methods of argument which have developed in this field are not exact and certain, but they are applicable to a very wide range of problems quite beyond the reach of more formal procedures. 2

(C) When this is so, much more elaborate and far-reaching deductions become possible because of the great power of mathematical notation and methods, which permit deductions to be made that would be overwhelmingly complex if argued in ordinary language. 0


① A-C-B ② B-A-C ③ B-C-A ④ C-A-B ⑤ C-B-A

 


125. 20수완 T3-36

 

As we trap more heat in the atmosphere, weather patterns become more volatile.


(A) That 7 percent isn't uniform, though. The greater moisture capacity of the air means that water can be sucked out of one area and deposited in another. Moisture becomes more concentrated in a few times and places, leading to droughts in one area or one season, followed by torrential rains in another. 1

(B) And indeed, across the United States, while precipitation is up 6 percent, droughts are also up, and the amount of rain that falls in the hardest 1 percent of rainstorms is up 20 percent. There's more rain, concentrated in fewer places, with more left to dry out, and those that do get rain receive more intense rainstorms. 2

(C) A rise in temperature of 1 degree can translate into an increase in the most severe extremes by several degrees. And warmer au can move more moisture around more quickly. For every degree Celsius that the planet warms, the atmosphere can absorb 7 percent more moisture. 0


① A-C-B ② B-A-C ③ B-C-A ④ C-A-B ⑤ C-B-A

 


126. 20수완 T3-37

 

Personal adornment may reflect inner emotional states called mood.


(A) The culturally encouraged interpretation is that the redirection of eye movements required by line contrasts is symbolic of a dynamic inner state within the wearer. Thus, at least for Americans, contrasting line and color in costume can express exuberant mood to others and also reinforce the same mood in the wearer. 2

(B) It may also reinforce, disguise, or create mood. An individual caught up in a certain mood may wish to externalize it so it can be conveyed to and shared with others. Perhaps an individual feels light-hearted and energetic. 0

(C) In America, a culturally recognized way to create this effect is to choose costume with colors and linear arrangements that show contrast rather than sameness. Typically, color contrast can be attained by using a number of different colors together, by using bright colors that contrast with a somber background, and by using sharp differences in lightness and darkness of color. Linear contrast occurs if lines suddenly change direction or intersect when proceeding in different directions. 1


① A-C-B ② B-A-C ③ B-C-A ④ C-A-B ⑤ C-B-A

 


127. 20수완 T3-38

 

The most striking characteristic of state public health law — and the one that underlies many of its defects — is its overall antiquity.


(A) When many of these statutes were written, public health (eg, epidemiology and biostatistics) and behavioral (eg, client-centered counseling) sciences were in their infancy. Modem prevention and treatment methods did not exist. 2

(B) These laws often do not reflect contemporary scientific understandings of injury and disease (eg, surveillance, prevention, and response) or legal norms for protection of individual rights. Rather, public health laws use scientific and legal standards that prevailed when they were enacted. Society faces different sorts of risks today and deploys different methods of assessment and intervention. 1

(C) Certainly, some statutes are relatively recent in origin. However, much of public health law was framed in the late nineteenth and early to mid-twentieth centuries and contains elements that are 40 to 100 years old. Old public health statutes are often outmoded in ways that directly reduce their effectiveness and conformity with modern standards. 0


① A-C-B ② B-A-C ③ B-C-A ④ C-A-B ⑤ C-B-A

 


128. 20수완 T3-39

 

The lesson of history is that civilizations have never learned to live in harmony with their environment.


(A) Instead, by depleting natural resources, they eventually reached a point where they could no longer maintain their civil and military infrastructure adequately. At that point, they became vulnerable to invaders who wiped them out. This pattern seems to have been the case for the Mayans, for Angkor and for Sumeria. 2

(B) There is abundant historical evidence to show that disregard for the environment leads to human disaster. The current furore over climate change is reminiscent of events within many doomed societies. 0

(C) Archaeological research shows that many, perhaps most, ancient civilizations destroyed themselves by degrading their environment. In most cases this did not mean that people all suddenly died of hunger or were consumed by storms. 1


① A-C-B ② B-A-C ③ B-C-A ④ C-A-B ⑤ C-B-A

 


129. 20수완 T3-40

 

In the few studies indicating somewhat favorable outcomes for aggressive humor, we note that mildly aggressive humor, when enacted as teasing or ridicule, may serve as a means of delivering certain types of messages.


(A) Participants viewed one of three videos (ie other-ridicule, self-ridicule, or no-target humor) and then performed different tasks to assess fear of failure, conformity, and creativity. Compared to participants who viewed the self-ridicule or no-target humor video, those in the other-ridicule condition were more conforming in completing their tasks and indicated a greater fear of failure. 2

(B) For example, research by Janes and Olsen examined young adults' perceptions of peer teasing and ridicule, which they labeled "jeer pressure." 0

(C) The researchers studied whether observing peers being ridiculed and teased in a video was related to college students' fear of failure, creativity, and conformity. 1


① A-C-B ② B-A-C ③ B-C-A ④ C-A-B ⑤ C-B-A

 


130. 20수완 T3-4142

 

Rather than offering suggestions to your students, offer them questions instead.


(A) By responding with questions you avoid being prescriptive. Asking a student questions about his writing, as would an honest reader attempting to understand the student's point of view, allows you to indicate places where his logic breaks down, to inquire about missing data or ideas, or to register confusion. Asking questions doesn't fix the broken logic, fill in the missing data, or clear up the writing that's confused you. It's up to the student to do these things as he adjusts his writing in response to your questions. 0

(B) We should read as though we are coming to the topic afresh, with no more knowledge than our students present us in their work. We should allow ourselves to become confused when a student's writing wanders and to lose our way when that writing is unclear. Challenge the student to understand why it is you are confused and to confront that confusion by adjusting her explanation. After all, it's her task to explain her meaning to you, and should you lose yourself in her arguments it's up to her to help you find your way out. 2

(C) Instead of showing the student how to fix his work, you've challenged him to find a way to fix it himself. Admittedly, it can be hard for us to respond as the honest readers for whom our students intend to write. We're considerably more knowledgeable about our disciplines than our students are, so it's easy for us to pave over the holes in our students ' logic without them having to do it for us. Ideally we should resist the urge to read their writing this way. 1


① A-C-B ② B-A-C ③ B-C-A ④ C-A-B ⑤ C-B-A

 


131. 20수완 T3-4345

 

Around the turn of the 20th century, when X-rays and ultraviolet light were still brand-new discoveries, the French academician Rene-Prosper Blondlot announced that he too had discovered a new kind of radiation.


(A) Then the lights came back on. In the front row, Wood sat wordlessly, holding up the aluminum prism for everyone to see. He had secretly removed the prism in the middle of Blondlot's demonstration to show that N-rays were an illusion. Without the prism, the machine couldn't work. After all, N-rays turned out to be fictive. The story of Blondlot is a story of self-deception among scientists. All scientists can fall into various forms of self-deception during research. To prevent self-deception, scientists need a strong commitment to carefulness, skepticism, and rigor. 2

(B) He visited Blondlot's laboratory. Blondlot attempted to demonstrate the existence of N-rays and make clear to Wood some of the difficulties involved. Blondlot's main apparatus was an N-ray spectroscope he had constructed with lenses and a prism all made of aluminum instead of glass. Blondlot would turn a dial to rotate the prism while his assistant read off the intensity of the N-ray beam focused on a screen. Time and again Blondlot rotated the prism and the assistant read off wavelengths of the transmitted beam. The emission spectrum seemed to be completely reproducible. As a final confirmation, and to make the measurements even more precise, Blondlot repeated turning the dial with the lights turned off, again with identical results. 1

(C) He called the new type of radiation N-rays, after Nancy, the name of the university where he worked. Within three years hundreds of papers had been written about N-rays, and he was awarded the prestigious Prix Leconte by the French Academy of Sciences. However, the problem was that almost no one outside Blondlot's laboratory could detect the rays at all. The problem, Blondlot explained, was that N-rays were sensitive to a thousand influences of every kind. Inexperienced workers could hardly hope to produce or detect them. Nature magazine was skeptical of his and sent American physicist Robert W. Wood of Johns Hopkins University to investigate his discovery. Wood suspected that N-rays were an illusion. 0


① A-C-B ② B-A-C ③ B-C-A ④ C-A-B ⑤ C-B-A

 


132. 20수완 T4-18

 

We are residents of the City of Racine and we support the Kenosha-Racine-Milwaukee (KRM) commuter rail.


(A) We welcome the KRM because it would make it easy for us to commute from Racine to Chicago and then to O ' Hare Airport, without getting in our car. It would also allow us to get to the Milwaukee Airport as well. 0

(B) Commuter rail will play an important part in contributing to economic development. We also have to be very sensitive to the environmental impact of all the automobiles and congestion in the region. Commuter rail, again, will have a positive impact on the environment. 2

(C) We are somewhat regular flyers and leaving our car at the airport can be inconvenient, time consuming and expensive. It would seem quite clear that the Chicago-Milwaukee Corridor represents the economic future for the region. 1


① A-C-B ② B-A-C ③ B-C-A ④ C-A-B ⑤ C-B-A

 


133. 20수완 T4-19

 

Ingrid traveled with her friend to Qinghai Province, to her hometown.


(A) These animals must not have learned to be afraid of humans. They watched them curiously, and it was not until they were very close that the animals trotted away; some even galloped along as if keeping them company. The sky was so blue that it seemed to have absorbed all the oceans in the world. 1

(B) Her uncle, a truck driver, drove them around. For a long time, their truck cruised on the desert-like land, no people or houses in sight. They saw many deer and wild goats, as well as coyotes. 0

(C) They arrived at Qinghai Lake. The huge prairie around it extended all the way to the horizon. She and her friend ran, jumped, sang, danced, rolled on the grass, and shouted at the lake. 2


① A-C-B ② B-A-C ③ B-C-A ④ C-A-B ⑤ C-B-A

 


134. 20수완 T4-20

 

One of the first questions I' m asked by adults who are interested in teaching philosophy to children is: "How will I know what material is age-appropriate for the kids I'm working with?"


(A) I believe that any topic can be discussed once you determine the general maturity level of your group and shift gears as needed. 2

(B) Interestingly, I've found there is no guarantee that one approach or presentation will work for a particular age group, and that it's best to trust your intuition about whether or how to approach a topic with your class or child. I have met second-graders who discussed death with ease, and seventh-graders who were uncomfortable talking about friendship. 0

(C) It's important not to assume immediately that a topic is appropriate or inappropriate on the basis of a child's age. Children's maturity levels, interests, backgrounds, and verbal abilities vary. 1


① A-C-B ② B-A-C ③ B-C-A ④ C-A-B ⑤ C-B-A

 


135. 20수완 T4-21

 

In his bright blue postman's uniform with gold buttons, Vincent's new friend, Joseph Roulin, sat stiffly posing for his portrait.


(A) Vincent thought him more interesting than anyone he'd met in Arles. When he finished the portrait, the Roulins invited him to stay for supper. Roulin, after a bottle of wine, expounded on his socialist politics and offered the younger artist advice about life. 1

(B) Vincent thought the postman looked like Socrates, with his large head, ruddy cheeks, and long salt-and-pepper beard. He painted quickly, as Roulin could hardly contain himself. Roulin's wife had just delivered a baby girl, and he was "proud as a peacock and aglow with satisfaction." 0

(C) For Vincent, who long ago had lost faith in his own father, Roulin, "so wise and so trustful," became a father figure. He painted eight versions of the postman, as well as portraits of Mrs. Roulin and their two sons. Despite his poverty, Roulin refused to be paid, so Vincent ended up buying him food and many drinks at the local cafe. 2


① A-C-B ② B-A-C ③ B-C-A ④ C-A-B ⑤ C-B-A

 


136. 20수완 T4-22

 

Wherever we find creativity, we almost always find it was the result of a person who willingly went to work on a real problem.


(A) Thomas Edison once remarked that "Everything comes to him who hustles." Work. Don't worry. 0

(B) When we read the words of people like Giacomo Puccini, that great operatic composer, who once remarked, "The music of this opera Madame Butterfly was dictated to me by God. I was merely instrumental in putting it on paper and communicating it to the public," what are we to think? Obviously, he and others feel as though they are merely the instrument through which creative energies are flowing. While it might have felt this way to Puccini, it is also evident that Puccini underestimated his own abilities.2

(C) That was Edison's advice. And he proved its usefulness by his own example. But despite Edison's experience and that of countless others who continue to make breakthroughs, there remains considerable mystery about how creative ideas actually come to people. 1


① A-C-B ② B-A-C ③ B-C-A ④ C-A-B ⑤ C-B-A

 


137. 20수완 T4-23

 

Placing organic products into the global market has a number of implications.


(A) Entry into global markets may offer grounds on which to challenge national subsidies for conventional agriculture, but retaliatory challenges against organic farming are likely. A further concern is that global markets are uncertain and often volatile, which has the effect of reducing the security of farming enterprises and can be added to the economic incentives for larger-scale enterprises. 2

(B) Global markets are characterized by the strong role played by corporations in transport, handling, distribution, marketing and sales. Entering into the same markets as conventional agricultural products is likely to result in organic produce being subject to the same economic conditions that have shaped conventional agriculture and made sustainable practices unattractive. 0

(C) Organic producers competing in existing global markets will face economic incentives likely to erode the principles of organic farming. An emerging issue of potentially great concern is challenges brought against nations whose trading preferences run counter to such groups as the World Trade Organization. 1


① A-C-B ② B-A-C ③ B-C-A ④ C-A-B ⑤ C-B-A

 


138. 20수완 T4-24

 

Speaking of the place of rational thinking in the life of primitive humanity, rationality itself meant more than mechanical, conscious figuring; it also expressed itself in different ways.


(A) Conscious, rational thinking can be an exceptionally sluggish and dangerous process in emergency situations, where split-second decisions often make the difference between survival and annihilation. 0

(B) It's not clear, however, whether the capacity for instant but unconscious thinking and split-second decision-making (eg, quickly running from a predator or thoughtlessly striking a debilitating or lethal blow to an enemy) is a new human response mechanism, an old instinct that has been carried over from humanity's animal ancestry, or a creative combination of the two. Whatever it is, it certainly realizes an important goal of rational figuring, to the extent that it is typically an orderly response that increases the chances for survival. 2

(C) It is for this reason that the capacity for rapid and dramatic decision-making had to evolve, which, oddly enough, was accomplished through bypassing conscious, rational thinking altogether. 1


① A-C-B ② B-A-C ③ B-C-A ④ C-A-B ⑤ C-B-A

 


139. 20수완 T4-26

 

Little Sable Point Light stands 107 feet tall in Silver Lake State Park.


(A) Although no longer manned, the tower still sends its light out into the night. The lighthouse is open to the public on weekends during the summer.2

(B) It is a simple lighthouse that reflects the tall cylindrical structure so common to Michigan lighthouses. It is one of the oldest brick lighthouses on Lake Michigan. Its red brick tower is topped by a black cast-iron parapet with arc-shaped supports. 0

(C) In 1874 it was considered one of the loneliest stations for a keeper because there was no village nearby. But, for anyone seeking solitude and space in the middle of unsurpassed beauty it was an ideal assignment. The keeper's house was demolished during the 1950s. 1


① A-C-B ② B-A-C ③ B-C-A ④ C-A-B ⑤ C-B-A

 


140. 20수완 T4-29

 

The artist is often unable to tell beforehand how his painting is going to turn out, because it develops as he paints it.


(A) But the medium has limitations of its own. There are feelings that can or cannot be translated effectively into a particular medium. In considering these limitations, the artist must realize how he changes the material and also in what ways the material changes his expression and his conceptions. 2

(B) The artist realizes what the layman has experienced but cannot express. The artist frees original fantasies, aims, desires, and emotions, and expresses them through his medium. 1

(C) Even with a particular compositional or subject matter in mind the design of a painting changes as one form is added to complement another and as these forms create new and unanticipated relationships when seen in a whole composition. A painting grows like a living organism in the artist's mind and feelings as he interacts with his work at different stages in its development. 0


① A-C-B ② B-A-C ③ B-C-A ④ C-A-B ⑤ C-B-A

 


141. 20수완 T4-30

 

Policies aimed at providing mobility for low-income carless people might effectively be focused on the census tracts that have the largest percentages of households with these characteristics.


(A) Even when an area-targeted policy can be implemented, it provides services to many households who live in the targeted tracts but do have a car or are not in poverty, and it would miss the many carless households that do not live in the target census tracts. 1

(B) Also, there are numerous individuals (rather than households) who are carless for much of the day — people, for example, who remain at home while someone else takes the household's one car to work. The census tract maps are little help in locating these people. 2

(C) You can see that such policies would be far easier to implement in the city, where target tracts are clustered together, than in the suburbs, where they are widely dispersed. 0


① A-C-B ② B-A-C ③ B-C-A ④ C-A-B ⑤ C-B-A

 


142. 20수완 T4-31

 

Probably the easiest way to tum your passion into your job is to do it gradually.


(A) If possible, don't quit your day job before launching your business. If you want to podcast, start with a monthly show or a very short weekly show and see how it goes. 0

(B) In many cases, if your idea is a good one, you'll eventually become so busy or successful in your part-time endeavor that it will be clear when you should quit your day job and become a full -time independent business owner. 2

(C) If you want to open a yogurt shop, take a part-time job in someone else's yogurt shop and learn everything you can about how to run the business. If you're passionate enough, it won't even feel like work. 1


① A-C-B ② B-A-C ③ B-C-A ④ C-A-B ⑤ C-B-A

 


143. 20수완 T4-32

 

When a class of 30 college students read a not very- good biography about Maria Tallchief, an Osage ballerina who captured the attention of the dancing world in the early 20th century, all the students, except for five women, pronounced the book "mediocre."


(A) The others, without ballet backgrounds, did not find enough to interest them in the shallow way the author presented Tallchief's story. 2

(B) When they read about Maria Tallchief, they were reading their own stories. For them, the book served as a link to a meaningful personal experience. 1

(C) That enthusiastic handful loved the book and couldn't understand why the others were not impressed by this story that had meant so much to them. During the short discussion, the fact surfaced that all of the five young women had taken and loved ballet as children. 0


① A-C-B ② B-A-C ③ B-C-A ④ C-A-B ⑤ C-B-A

 


144. 20수완 T4-33

 

Many have claimed that, with billions of likely Earth-like planets, civilizations like ours must be common in our galaxy.


(A) More fundamentally, complex biological beings did not evolve to traverse the vastness of interstellar space; if interstellar travelers exist they'll be robots capable of "sleeping" over many thousands of years. Remember that stars are separated in distances measured by "light years" and, with light speed at 186,000 miles (300,000 km) in a single second, interstellar travel by living things remains a fantasy. 1

(B) Setting astrobiology ("the science without a subject") aside we'll confine ourselves to the planet on which we live. 2

(C) However, the more we learn, the more unlikely that appears. SETI — the Search for Extraterrestrial Intelligence program 一has been scanning the firmament for radio signals over more than forty years, and they have failed to intercept a single coherent message. 0


① A-C-B ② B-A-C ③ B-C-A ④ C-A-B ⑤ C-B-A

 


145. 20수완 T4-34

 

When someone gives a reason for his action, he makes reference to an end (or goal), plus a belief that the action will somehow advance the end.


(A) Humans assign variable meanings to what they perceive, dependent on perception, belief, emotion, and long-term goals. Knight notes that human consciousness does not fit neatly into the natural science paradigm. "Science can find no place for it[consciousness], and no role for it to perform in the causal sequence. 1

(B) The very invocation of a reason for action suggests that the person himself is a cause, directing himself toward an end. Economist Frank Knight addresses the problem of reasons for scientific accounts of choice as part of the more general problem of the place of human consciousness in scientific explanation. The challenge of human consciousness arises from the fact that humans react to their interpretation of reality, not to reality directly. 0

(C) It is epiphenomenal." The nature of social science, and its relationship to natural science, depends crucially on whether or not we take seriously the reasons that people give as causes of their actions. 2


① A-C-B ② B-A-C ③ B-C-A ④ C-A-B ⑤ C-B-A

 


146. 20수완 T4-35

 

It is time to put agriculture and food on the political agendas of local communities.


(A) This integration means that local agriculture and food businesses need the same access to economic development resources — such as grants, tax incentives, and loans — as nonfarm-related businesses. 1

(B) Additionally, communities should ensure that agricultural constituencies are represented on community boards, task forces, and governing bodies. Likewise, local agriculture and food systems activities should be addressed and integrated into any comprehensive planning processes. 2

(C) Locally organized agriculture and food enterprises must be fully integrated into a community's general planning and economic development efforts. 0


① A-C-B ② B-A-C ③ B-C-A ④ C-A-B ⑤ C-B-A

 


147. 20수완 T4-36

 

Increased daytime temperature of both pavement and near-surface air, reduced nighttime cooling, and associated higher air pollution levels can affect human health by contributing to general discomfort, respiratory difficulties, heat cramps and exhaustion, nonfatal heat stroke, and heat-related mortality.


(A) Excessive heat events, or abrupt and dramatic temperature increases, are particularly dangerous and contribute to premature deaths, potentially resulting in above-average rates of mortality. The heat-related mortality may tend to increase under the future potential climate change, under which more extreme surface weather conditions and heat events will probably be showing up. 2

(B) Sensitive populations, such as children, older adults, and those with existing health conditions, are at particular risk from these events. 1

(C) Heat islands can also exacerbate the impact of heat waves, which are periods of abnormally hot, and often humid, weather. 0


① A-C-B ② B-A-C ③ B-C-A ④ C-A-B ⑤ C-B-A

 


148. 20수완 T4-37

 

It is important to recognize the nature of mathematics and the very radical abstraction that it involves.


(A) Escaping from the earlier era of crude superstition and magic, they saw themselves emerging into a world where reason and evidence would triumph. But reason, in its purest form, is seen in logic and mathematics, and it was therefore natural to expect that the world would be, in principle, comprehensible in terms of 'laws of nature' which, with mathematical precision, would determine the movement of all things. 2

(B) That it should be possible for an abstract formula to correspond to nature was a fundamental assumption made by those involved in the emerging sciences. Beneath it lay the deeper assumption that the world is a predictable and ordered place. 1

(C) Galileo, Descartes, Huygens and Newton all produced formulae. In other words, they were seeking to create a mathematical and abstract way of summing up physical phenomena, using mathematics to express patterns seen in nature. 0


① A-C-B ② B-A-C ③ B-C-A ④ C-A-B ⑤ C-B-A

 


149. 20수완 T4-38

 

An interesting observation that is pertinent to the idea of an instinctive urge is displacement activity.


(A) But on other occasions the behavior is not distinguishable from normal grooming activity. Certain species of fish, such as the stickleback, also exhibit such out-of-context displacement activity. When at the boundary between its own territory and that of another stickleback, where both attack and escape behaviors are elicited, inappropriate nest-building behavior is often displayed. 2

(B) Displacement activity occurs when two incompatible response tendencies are simultaneously aroused. For example, a bird might be faced with a rival that elicits both attack and flight. 0

(C) In this situation, behaviors are displayed that appear to be irrelevant to the situation, such as grooming. At times the activity differs from normal grooming behavior in that it seems hurried and is discontinued before it is completed. 1


① A-C-B ② B-A-C ③ B-C-A ④ C-A-B ⑤ C-B-A

 


150. 20수완 T4-39

 

As the number of information security incidents rose, a community of security practitioners and academics attempted to categorize the types of threats.


(A) This is an incredibly problematic term especially when it is employed outside the computer and information security community. We all know a military attack or a terrorist attack is an event that carries with it the likely loss of life and destruction of property, but what the policy crowd didn't have a firm grasp upon was what sort of damage a cyber attack could do. 2

(B) Into these categories, supplemented by a few others that often are subject to debate, almost any security can be placed. In computer and information security, any "single unauthorized access attempt, or unauthorized use attempt, regardless of success" carries the label of "attack," a word with many applications in the English language, from verbal dispute to acts of war. 1

(C) While the US military had considerable experience in how it identified, classified, protected, transmitted, processed, and destroyed information, outside of government such knowledge was scarcer. Study of security produced a characterization of three categories in which information or information systems could be compromised: confidentiality, integrity, and availability. 0


① A-C-B ② B-A-C ③ B-C-A ④ C-A-B ⑤ C-B-A

 


151. 20수완 T4-40

 

Researchers have now integrated findings that used methods such as positron emission tomography (PET) imaging studies of dopamine's role in human drug addiction and longitudinal observations of the eating behaviors of overweight and obese individuals to show that increased body weight can increase behaviors that are common markers of addiction.


(A) By hiring food and flavor chemists to design products that elicit this primal "wanting," food companies can bypass the rational processes that protect people from harming themselves. 2

(B) Dopamine rewards the brain, thus signaling people to continue the behaviors that release it. One characteristic of addiction is that it is experienced as wanting, not simply liking, the addictive substance. 0

(C) A biopsychological research group in the United Kingdom has developed experimental methods to distinguish between explicit liking and implicit wanting. Their studies found that individuals with a tendency towards binge eating reported "liking" most food types, but "wanting" high-fat sweet foods. 1


① A-C-B ② B-A-C ③ B-C-A ④ C-A-B ⑤ C-B-A

 


152. 20수완 T4-4142

 

There is a distinction between the social and physical environments and the different types of selection pressures that they represent; this is because the physical environment changes a lot less over evolutionary time than the social.


(A) It is much better if behavioural responses are flexible. Many human brain adaptations have evolved in response to aspects of the social environment, including cheating, deception, and cooperation. 2

(B) For example, the qualities of gravity feel no different to you and me than they did to the first land-dwelling vertebrates. This stability means that there are certain properties of tetrapod bone structure, including in us four-limbed humans, that are set and have not changed. Tetrapods have a good solution to the problem of bearing weight under gravity, and any mutations are likely to result in a lower survival and reproductive outcome (a lower fitness). 0

(C) The social environment — made up primarily of other humans, but also more broadly other animals — is a completely different matter. In this environment, humans have to react to situations as they occur. Here, doing the same thing every time a situation occurs — say, always dodging left when being charged by a bull, or always sharing your meat when asked to 一 would be a distinct disadvantage, because the other animals or humans would evolve a response to outwit you. 1


① A-C-B ② B-A-C ③ B-C-A ④ C-A-B ⑤ C-B-A

 


153. 20수완 T4-4345

 

'Granddad,' William shouted, still dazed.


(A) William, not wanting to be left on his own, had followed his grandfather up to his room, and entered in time to see and hear him utter those unusual words. To his astonishment and delight, he saw the shiny box turn red, as if it had suddenly become very hot, and then the lid opened and revealed the most strikingly beautiful ring imaginable. 'Granddad' shouted William excitedly. 'Is that the ring of ever-changing colours you are always telling me about? It's so — so cool,' he said, not wanting to use a girly word like beautiful, although he thought it was. He came closer to get a better look at it, and to his amazement it began changing colours. First it went green, then blue, then pink and then through every colour in the spectrum. 1

(B) 'What's wrong?' His grandfather didn't answer — he probably never heard him, his mind wrestling with his worst fears. Granddad rushed into his room, flung back the door to his cupboard and searched through a pile of old shoeboxes. Hidden inside one of them was a smaller box, which he removed carefully. He unhooked the little catch at the front and lifted the lid. Inside was an even smaller box made of what looked like stainless steel. He lifted the box out, took it over to his desk and said, 'Ism Sprism.' 0

(C) Just when William thought there couldn't possibly be any more colours, new ones he had never seen before started appearing. 'Those are magic colours,' said his grandfather. They watched, fascinated. For a moment, his grandfather even forgot the reason for rushing to his room. 'Then you really are a wizard after all' shouted William, shattering the calm around them. A shiver of amusement ran down his spine. Before his grandfather could answer, a light exuded from the ring which was so deep and vibrant that William thought he would drown in it. 2


① A-C-B ② B-A-C ③ B-C-A ④ C-A-B ⑤ C-B-A

 


154. 20수완 T5-18

 

I reside in the downtown area of Louisville, close to the proposed parking garage location.


(A) In addition, the proposed size is out of character for downtown in general. Even a three-story parking garage would be the largest building in this area, damaging the historic character of the neighborhood and looming over the skyline. 1

(B) As an active resident who spends time downtown on most Friday and Saturday nights, the only times there seems to be a true parking problem are during the Summer Concerts in July. Building a four-story parking garage for those eight nights seems to be a bit extreme. 0

(C) I think there are other projects that I'd prefer the city to prioritize before undertaking a new parking garage. 2


① A-C-B ② B-A-C ③ B-C-A ④ C-A-B ⑤ C-B-A

 


155. 20수완 T5-19

 

When I reached the intersection, Sheldon was not there.


(A) Could I be in the wrong place? Perspiration began to form on my forehead and I wiped away the dampness on my upper lip. I forced myself to control my fear, and tried to think of a plan. If I called Sheldon there would be no answer, as he was waiting for me on a street corner somewhere nearby. It might be a good idea to drive closer to his office. 1

(B) Fortunately, I had his business card in my wallet, because I simply could not remember his address. Then, I spotted him standing on the corner. Concerned, he had walked down the block. Thank goodness! 2

(C) Where was he? Cars behind me began to honk. I had to drive on. There was no place to park. Again I passed the intersection and no Sheldon. 0


① A-C-B ② B-A-C ③ B-C-A ④ C-A-B ⑤ C-B-A

 


156. 20수완 T5-20

 

Collection reevaluation, most commonly known as weeding, is the process of removing books and other materials from the library collection when they are no longer useful.


(A) Instead, it should be motivated by a desire to maintain an up-to-date, inviting, and useful collection. Just as weeds can overtake a garden and hide the beauty of flowers, materials left in the library collection which are no longer useful can make it more difficult to find the ones which are. 2

(B) It is an essential part of collection development. A good weeding policy and procedure is as important as good selection procedures in building and maintaining an attractive and useful collection. 0

(C) Because it is a task many librarians don't enjoy, weeding is often put off until space is needed. However, overcrowded shelves should not be the first or only motivation for weeding. 1


① A-C-B ② B-A-C ③ B-C-A ④ C-A-B ⑤ C-B-A

 


157. 20수완 T5-21

 

Ideas about uncertainty are governed by the way society perceives the relationship between the present and the future.


(A) When, as today, the future is regarded as a dangerous territory, uncertainty is framed in a negative light. In such a setting, change itself is perceived as threatening. A potent undercurrent of apprehension towards change — whether technological, social or political — permeates the day-to-day affairs of the contemporary Western world. 0

(B) In this way, the dangers of the future acquire an immediate and intimate quality. They demand that we ring the alarm bells while implying that there is very little that can be done to avoid the dangers that lie ahead. 2

(C) Uncertainty was at times regarded as an opportunity — that it now tends to be cast in a negative light is symptomatic of a mood of fatalism towards the challenges faced by society. This fatalistic attitude is summed up by the often-repeated catch-phrase — 'The question is "not if, but when?"' Warnings of catastrophic climate events, deadly flu epidemics or mass casualty terrorism usually conclude with this defeatist refrain, which implicitly and sometimes explicitly calls into question humanity's capacity to avoid the destructive consequences of the threats it faces. 1


① A-C-B ② B-A-C ③ B-C-A ④ C-A-B ⑤ C-B-A

 


158. 20수완 T5-22

 

At a recent workshop for mental health professionals, Bonnie Zucker, the author of two excellent books on childhood anxiety, gave a presentation on the treatment of anxiety.


(A) Dr. Zucker then said, "Meditation is so powerful that I ask all of you who don't yet meditate to learn meditation — and then call me in a year to tell me how it's changed your life." We couldn't agree more. 1

(B) She asked the three hundred professionals in the audience if they meditated on a regular basis. A handful raised their hands. 0

(C) Practicing meditation is increasingly important as changes in the world lead to higher levels of anger and fear, and as advances in technology quicken the pace of life, giving us little time to simply "be" with ourselves. Although kids and teens rarely beg their parents to find them a meditation teacher, research indicates that when children and adolescents establish a practice regularly, meditation benefits them in the same ways as it does adults. 2


① A-C-B ② B-A-C ③ B-C-A ④ C-A-B ⑤ C-B-A

 


159. 20수완 T5-23

 

In times of crisis, the media react to society's need for surveillance and information by devoting massive time and energy to coverage of the crisis.


(A) The media highlight the wisdom of leaders and the bravery of rescue workers or soldiers to reassure society that "we are all in this together" and that everything possible is being done for survival. So, although the media may be unable to fulfill surveillance and correlation needs, they are able to offer assurance and tension reduction. 2

(B) All too often, though, it is difficult to gather information. Yet, it would be dysfunctional for media coverage to cease until information can be collected and verified. 0

(C) In order to reduce tension in society, media devote a good deal of coverage to media content intended to comfort their audience. Solidarity building is functional for society in times of crisis. 1


① A-C-B ② B-A-C ③ B-C-A ④ C-A-B ⑤ C-B-A

 


160. 20수완 T5-24

 

Affect can be expressed through facial expressions and through modulations in the tone and prosody of the voice.


(A) What is striking is the finding that the input from the body — including signals from the muscles, bones, and viscera (such as the heart and the intestines) — is more highly integrated in the right hemisphere than in the left. In other words, the whole body is represented in an integrated way in the right hemisphere. 1

(B) Even the regulation of the body's autonomic nervous system is primarily mediated by right-brain mechanisms. The right hemisphere therefore appears to play a major role in mediating regulatory emotional processes, as well as in permitting the expression of emotional states and the conscious awareness of emotional experience. 2

(C) These nonverbal aspects of language communication, in both their expression and perception, appear to be mediated predominantly by the right hemisphere. The body's posture and movement can also blend with the voice and facial expression in sending affective signals that are readily perceived by other people. 0


① A-C-B ② B-A-C ③ B-C-A ④ C-A-B ⑤ C-B-A

 


161. 20수완 T5-25

 

The above graph shows the online market shares of eight European countries in 2014 and 2015.


(A) In 2015, the Netherlands' online market share was more than twice as large as Spain's. 2

(B) The online market shares of the eight European countries all increased from 2014 to 2015. Both in 2014 and in 2015, the country with the largest online market share was the United Kingdom, followed by Germany. 0

(C) Sweden took third place in the online market share in 2014, but it fell to fourth place behind France in 2015. Spain's online market share was 0.5 percentage point larger in 2015 than it was in 2014. 1


① A-C-B ② B-A-C ③ B-C-A ④ C-A-B ⑤ C-B-A

 


162. 20수완 T5-29

 

At the root of the task of epistemology is the challenge posed by the simple fact that appearances don't always correspond with reality.


(A) Sherlock Holmes seemed to have an acute recognition of this insight, perhaps accounting for what can be called his aversion to the obvious. Rarely was he content with what may have seemed clear-cut and obvious to others. He was interested in what accounted for all the facts, not just those most at the surface. 1

(B) An obvious explanation in "The Beryl Coronet" case was that the son was the guilty perpetrator, appearing to have been caught red-handed. Holmes, searching for an explanation for all the observations he had made and facts in need of an account, remained skeptical. Healthy skepticism about appearances tends to be a salient feature of any credible epistemologist. 2

(C) They can be deceiving. Sometimes what seems simple is deceptively complex, and sometimes what appears complicated admits of a simple explanation. The fact that we can be deceived or deluded complicates the epistemic task of finding the truth. 0


① A-C-B ② B-A-C ③ B-C-A ④ C-A-B ⑤ C-B-A

 


163. 20수완 T5-30

 

For some teens, screen use has contributed to an underdeveloped identity.


(A) They may be devoting so much time to gaming and staying connected with "friends" that they don't have time or desire to broaden their interests or learn new skills, which would grow and solidify their identity. Also, because technology makes many things easier, they may be lacking the perseverance, diligence, and teachability that are often essential for adding to their skill sets and character development. 0

(B) For example, they may post often about their musical interests and never mention that they volunteer at an animal shelter. As time goes by and no friends acknowledge or support the teen's interest in animal rescue, the teen may begin to devalue that interest and let it wane. 2

(C) Posting on social networks can limit identity development because the tendency is for posts to acknowledge only certain aspects of their lives. That means friends and family can only provide comments or ask questions about those elements — the ones the teens choose to show. 1


① A-C-B ② B-A-C ③ B-C-A ④ C-A-B ⑤ C-B-A

 


164. 20수완 T5-31

 

Although both papyrus and parchment were used in various places throughout the ancient world for centuries, the eventual displacement of papyrus as the standard writing medium was fueled by a rivalry.


(A) As Alexandria was accumulating volumes and copies of documents, so was the library in Pergamum. Another important center of scholarship, the Greek kingdom of Pergarnum was located in present-day Turkey and was founded in the second century B.C. 0

(B) By the waning of the Western Roman Empire in 476 A.D., the parchment codex had become the predominant form of the book, replacing the papyrus roll. 2

(C) Not to be outdone by a competitor, Egypt placed an embargo on the export of the native papyrus, thus preventing Pergamum from obtaining the needed writing medium. Cutting off the supply of papyrus only led the scholars in Pergamum to improve parchment and to rely on it as their main writing material. 1


① A-C-B ② B-A-C ③ B-C-A ④ C-A-B ⑤ C-B-A

 


165. 20수완 T5-32

 

Technology now allows for addition as well as subtraction.


(A) We can not only imagine that the full assembly is present, but also prove it. 1

(B) At an annual gathering of my college friends in Vermont's Green Mountains, those unable to make it up for the weekend are photoshopped in by one friend; though absent, they are still sitting only slightly awkwardly on the steps or by the porch rail, reaffirming the inclusive spirit of our extended family. 0

(C) In one such photo taken several years ago, a man who happened to be in New York City that weekend appeared on the Vermont porch tossing a basketball to some hoop outside the frame. This magical realism was consistent with the way our long-standing friendships are integrated — seamlessly, unpredictably, and mysteriously — into geographically remote lives and experiences. 2


① A-C-B ② B-A-C ③ B-C-A ④ C-A-B ⑤ C-B-A

 


166. 20수완 T5-33

 

Most early primate studies assumed that primates were highly territorial and that groups would fight to defend their territories.


(A) Or just running away. Severe violence and aggression between groups is rare and seldom results in death. 2

(B) This is not to say that if one spends enough time watching primates she won't see two groups corning together over a contested area and putting on a big show for each other — lots of hooting and hollering and maybe even some fighting. These conflicts can result in serious injury or death but rarely do. Just as within groups, between-group conflicts are often resolved via negotiations or avoidance. 1

(C) We now know that most primates are not what we would call "territorial," because areas they use overlap with areas that other groups of the same species use. But there are conflicts over space, and in most cases groups of the same species tend to avoid being in the same place at the same time (though not always). Researchers have argued that this is a way to minimize the risk of conflict and violence between groups. 0


① A-C-B ② B-A-C ③ B-C-A ④ C-A-B ⑤ C-B-A

 


167. 20수완 T5-34

 

Of all approaches to health, the biomedical is unquestionably the most influential.


(A) There are obvious merits to understanding these physiological influences, not least to treat infectious diseases, which were the main cause of ill health and death until early in the 20th century. However, as Engel recognised, ill health is not reducible to disease processes alone, and if it were, then there should be much greater consistency in how people experience and respond to disease and its treatment than is actually observed. 1

(B) This model understands health primarily through the lens of disease, and it attributes the cause of ill health to some breakdown in normal biological and physiological functioning. In so doing, it gives a clear direction in how best to manage health — and this is to focus on repairing or treating the source of breakdown in the body. 0

(C) It is also the case that the health landscape has changed dramatically to one in which chronic conditions (eg, diabetes, depression, arthritis) have become the prevailing cause of ill health. For these conditions, there is generally no simple biomedical fix that can be administered to restore health.2


① A-C-B ② B-A-C ③ B-C-A ④ C-A-B ⑤ C-B-A

 


168. 20수완 T5-35

 

Most people are socialized to believe in the reciprocity principle — the rule that one should pay back in kind what one receives from others.


(A) Charities frequently make use of this principle. 0

(B) The belief that people should reciprocate others' kindness is a powerful norm; thus, people often feel obliged to reciprocate by making a donation in return for the gift. According to Cialdini, the reciprocity norm is so powerful that it often works even when the gift is uninvited, the gift comes from someone you dislike, or the gift results in an uneven exchange. 2

(C) Groups seeking donations for the disabled, the homeless, and so forth routinely send "free" address labels, key rings, and other small gifts with their pleas for donations. 1


① A-C-B ② B-A-C ③ B-C-A ④ C-A-B ⑤ C-B-A

 


169. 20수완 T5-36

 

Color defines our world and our emotions.


(A) Upon entering a room, we first see the color or colors used in the interior design and then discern the furnishings and artifacts contained within the space. An artwork, be it fine or commercial, is aesthetically pleasing to the viewer when its color usage allows the viewer to see the content of the piece (both color and imagery) together. When this is accomplished, a work's message is conveyed immediately, without a "second look" on the part of the viewer. 2

(B) At first glance we do not see the different species of trees present in a summer woodland, but rather see the preponderance of green. The artist, architect, and designer, however, are generally concerned with having color and imagery perceived simultaneously. 1

(C) It is usually seen before imagery. Our eyes are attracted to color to such an extent that the color of an object is perceived before the details imparted by its shapes and lines. 0


① A-C-B ② B-A-C ③ B-C-A ④ C-A-B ⑤ C-B-A

 


170. 20수완 T5-37

 

How the bandwagon effect occurs is demonstrated by the history of measurements of the speed of light.


(A) If a result fit what they expected, they kept it. If a result didn't fit, they threw it out. They weren't being intentionally dishonest, just influenced by the conventional wisdom. The pattern only changed when someone had the courage to report what was actually measured instead of what was expected. 2

(B) Then, from 1900 to 1950, the opposite happened — all the experiments found speeds that were too low! This kind of error, where results are always on one side of the real value, is called "bias." It probably happened because over time, experimenters subconsciously adjusted their results to match what they expected to find. 1

(C) Because this speed is the basis of the theory of relativity, it's one of the most frequently and carefully measured quantities in science. As far as we know, the speed hasn't changed over time. However, from 1870 to 1900, all the experiments found speeds that were too high. 0


① A-C-B ② B-A-C ③ B-C-A ④ C-A-B ⑤ C-B-A

 


171. 20수완 T5-38

 

Loess is a homogeneous, fine yellow soil that has been deposited across 1 million square miles (2 million sq km) of land that covers several areas of the world: Asia, Europe, and North America.


(A) It ranges in thickness from area to area and can be as thick as 10 feet (3 m) in some locations. Loess originated from glacial processes. 0

(B) As the area dried out, winds carried loess across the land. Its texture was so fine that it was carried great distances. This spread the deposits across wide areas and left rich, easily recognizable, homogeneous soil. 2

(C) As the massive weight of the glacial ice moved across the Earth's surface, the ice ground along the rock slowly and abraded and pulverized it into a powderlike substance. Later, as the climate warmed and the ice melted, running water washed the flourlike deposits from under the glaciers and into streams along the edges of the ice. 1


① A-C-B ② B-A-C ③ B-C-A ④ C-A-B ⑤ C-B-A

 


172. 20수완 T5-39

 

In one experiment, participants received verbal information about a product that they believed to be either familiar (a standard computer mouse) or unfamiliar (a trackball mouse).


(A) Consequently, visualizers evaluated the unfamiliar product less favorably than the familiar one. However, presenting a picture of the unfamiliar product increased visualizers' evaluation of it to a level similar to that of the familiar one. 1

(B) In contrast, individuals with a disposition to process information verbally (verbalizers) based their evaluations on the semantic implications of the product-attribute descriptions, which were the same regardless of the product's ostensible familiarity. Therefore, they evaluated the familiar and unfamiliar products similarly even in the absence of a picture. 2

(C) Individuals with a disposition to process information visually (visualizers) presumably had a previously formed mental image of the familiar product that they could use in organizing the verbal attribute descriptions. When the mouse was unfamiliar and a picture of it was unavailable, this was not the case, and a visual image of the product was difficult to construct on the basis of the attributes alone. 0


① A-C-B ② B-A-C ③ B-C-A ④ C-A-B ⑤ C-B-A

 


173. 20수완 T5-40

 

Richerson and Boyd argue that people could learn cooperative behavior through "conformist transmission," which means copying whatever behaviors are most common within a population.


(A) Herbert Simon has made a similar argument about our disposition to learn from others, which he calls "docility." This is a highly advantageous trait, yet it allows other people to occasionally manipulate us into learning altruistic behaviors which do not benefit us. 2

(B) Given that humans rely so much on socially learned behaviors, this bias toward conformist transmission is probably adaptive on average, even if it occasionally causes us to learn maladaptive behaviors. Thus, it is possible that our tendency to copy prosocial behavior could be a maladaptive byproduct of our adaptive tendency to copy common behaviors. 1

(C) Copying common behaviors is normally an adaptive way of learning how to do things because others may have already figured out the best way of doing things. However, it can also cause people to make systematic errors when they also copy behaviors that tum out to not be optimal. 0


① A-C-B ② B-A-C ③ B-C-A ④ C-A-B ⑤ C-B-A

 


174. 20수완 T5-4142

 

The last, most needed, most paradoxical attribute of democratic leadership is the willingness to forgo power when attaining and maintaining it requires morally unacceptable compromise.


(A) On the other hand, principled aspirants cannot hope to win power by bluntly saying exactly what they believe. For example, while his desire to support Britain's struggle against Nazi Germany was completely justified, Franklin Roosevelt might well have lost his 1940 re-election campaign if he had been completely candid about it. So he equivocated. When Wendell Willkie, the Republican presidential nominee, claimed that a vote for Roosevelt meant war in 1941, Roosevelt countered with a flat promise to the contrary — "Your boys are not going to be sent into any foreign wars" — deliberately omitting the Democratic platform's qualifying phrase, "except in case of attack." 1

(B) Democratic politics at its best is the use of publicly authorized power to advance the common good. Would-be leaders, then, can fail in two ways. They may be unable to obtain public support for their agenda, or they may win support by advocating only what the people want to hear. While modem survey research has raised the assessment of public beliefs to a high art, the temptation to pander to them is a recurrent weakness of democratic politics. 0

(C) When one of his speechwriters asked about the omission, he replied, "Of course we'll fight if we're attacked. If someone attacks us, then it isn't a foreign war, is it?" This mental reservation allowed Roosevelt to pretend that he wasn't trying to mislead the people, which of course he was. 2


① A-C-B ② B-A-C ③ B-C-A ④ C-A-B ⑤ C-B-A

 


175. 20수완 T5-4345

 

One Friday afternoon in July, an editor at Australian Football League (AFL) media called David, an intern reporter, and asked if he was interested in covering the game at the Melbourne Cricket Ground (MCG) that night (Collingwood versus Adelaide).


(A) He was still sitting in the MCG press box at 1 a.m. with a few other reporters, but it was all worth it in the end. David's experience at AFL media has, above all things, given him the best indication yet of what it takes to be a journalist at that level. Watching the experienced guys go about their work and being able to pick their brains at various times has been a really positive experience. David hopes his time there has given him a solid grounding for future work in sports journalism, an area he is most passionate about. 2

(B) One of the reporters suddenly quit her job and he was asked to take her spot. Naturally, he jumped at the chance and was assigned to sit in the press box and provide coverage of the game alongside one of the other AFL media reporters. Sitting with prominent football journalists in Melbourne was very exciting to David. As the night progressed he realised how ruthless match reporting could be. He was expected to file his report on or within five minutes of the final siren, which proved difficult given the match's relatively close result. 0

(C) After the game he went down to the losing team's rooms (Adelaide) and approached their media manager for a player interview. Before the game he had seen on Twitter that Adelaide player Jarryd Lyons — who kicked four goals on the night — had played school cricket with new Australian sensation Ashton Agar. Ashton Agar had just scored a brilliant 98 on debut in England the night before and David wanted to speak with Jarryd Lyons for a potential story. His request was instantly granted and he spoke to him, firstly about the game and eventually about playing with Agar at De La Salle College. He also attended Adelaide coach Brenton Sanderson's press conference and wrote a report on that. 1


① A-C-B ② B-A-C ③ B-C-A ④ C-A-B ⑤ C-B-A

 


[ANSWER]
1. ① 2. ④ 3. ③ 4. ③ 5. ② 6. ④ 7. ② 8. ⑤ 9. ③ 10. ①


11. ① 12. ① 13. ① 14. ① 15. ⑤ 16. ② 17. ④ 18. ③ 19. ① 20. ③


21. ④ 22. ③ 23. ④ 24. ④ 25. ② 26. ④ 27. ④ 28. ① 29. ① 30. ③


31. ① 32. ⑤ 33. ① 34. ④ 35. ① 36. ③ 37. ③ 38. ① 39. ② 40. ⑤


41. ③ 42. ④ 43. ② 44. ① 45. ① 46. ② 47. ① 48. ② 49. ① 50. ②


51. ② 52. ④ 53. ② 54. ② 55. ③ 56. ① 57. ② 58. ① 59. ③ 60. ③


61. ④ 62. ④ 63. ③ 64. ② 65. ③ 66. ③ 67. ① 68. ④ 69. ④ 70. ④


71. ④ 72. ③ 73. ① 74. ④ 75. ② 76. ④ 77. ④ 78. ② 79. ④ 80. ④


81. ⑤ 82. ⑤ 83. ② 84. ① 85. ⑤ 86. ⑤ 87. ② 88. ② 89. ② 90. ⑤


91. ① 92. ② 93. ① 94. ③ 95. ② 96. ③ 97. ③ 98. ① 99. ④ 100. ②


101. ① 102. ③ 103. ④ 104. ② 105. ③ 106. ③ 107. ② 108. ⑤ 109. ④ 110. ⑤


111. ⑤ 112. ② 113. ⑤ 114. ② 115. ⑤ 116. ⑤ 117. ⑤ 118. ④ 119. ③ 120. ⑤


121. ① 122. ③ 123. ① 124. ④ 125. ④ 126. ③ 127. ⑤ 128. ③ 129. ③ 130. ①


131. ⑤ 132. ① 133. ② 134. ③ 135. ② 136. ① 137. ③ 138. ① 139. ③ 140. ⑤


141. ④ 142. ① 143. ⑤ 144. ④ 145. ② 146. ④ 147. ⑤ 148. ⑤ 149. ③ 150. ⑤


151. ③ 152. ③ 153. ② 154. ② 155. ④ 156. ③ 157. ① 158. ② 159. ③ 160. ④


161. ③ 162. ④ 163. ① 164. ① 165. ② 166. ⑤ 167. ② 168. ① 169. ⑤ 170. ⑤


171. ① 172. ④ 173. ⑤ 174. ② 175. ③ 

728x90
반응형